Buy BOOKS at Discounted Price

Electrochemistry

Class 12th Chemistry NCERT Exemplar Solution
Multiple Choice Questions I
  1. Which cell will measure standard electrode potential of copper electrode?…
  2. Which cell will measure standard electrode potential of copper electrode?…
  3. Electrode potential for Mg electrode varies according to the equation. The graph of…
  4. Electrode potential for Mg electrode varies according to the equation. The graph of vs…
  5. Which of the following statement is correct?
  6. Which of the following statement is correct?
  7. The difference between the electrode potentials of two electrodes when no current is drawn…
  8. The difference between the electrode potentials of two electrodes when no current is drawn…
  9. Which of the following statement is not correct about an inert electrode in a cell?…
  10. Which of the following statement is not correct about an inert electrode in a cell?…
  11. An electrochemical cell can behave like an electrolytic cell when _________.…
  12. An electrochemical cell can behave like an electrolytic cell when ____________.…
  13. Which of the statements about solutions of electrolytes is not correct?…
  14. Which of the statements about solutions of electrolytes is not correct?…
  15. Using the data given below find out the strongest reducing agent.…
  16. Using the data given below find out the strongest reducing agent.…
  17. Use the data given in Q.8 and find out which of the following is the strongest oxidising…
  18. Use the data given in Q.8 and find out which of the following is the strongest oxidising…
  19. Using the data given in Q.8 find out in which option the order of reducing power is…
  20. Using the data given in Q.8 find out in which option the order of reducing power is…
  21. Use the data given in Q.8 and find out the most stable ion in its reduced form.…
  22. Use the data given in Q.8 and find out the most stable ion in its reduced form.…
  23. Use the data of Q.8 and find out the most stable oxidised species.…
  24. Use the data of Q.8 and find out the most stable oxidised species.…
  25. The quantity of charge required to obtain one mole of aluminium from Al2O3 is ___________.…
  26. The quantity of charge required to obtain one mole of aluminium from Al2O3 is ___________.…
  27. The cell constant of a conductivity cell _____________.
  28. The cell constant of a conductivity cell _____________.
  29. While charging the lead storage battery ______________.
  30. While charging the lead storage battery ______________.
  31. is equal to __________________.
  32. is equal to __________________.
  33. In the electrolysis of aqueous sodium chloride solution which of the half-cell reaction…
  34. In the electrolysis of aqueous sodium chloride solution which of the half-cell reaction…
  35. The positive value of the standard electrode potential of Cu2+/Cu indicates that…
  36. for some half cell reactions are given below. On the basis of these mark the correct…
  37. = 1.1 V for Daniel cell. Which of the following expressions are correct description of…
  38. Conductivity of an electrolytic solution depends on ____________.…
  39. lambda _{m}^{0}h_{2}o is equal to ___________.
  40. What will happen during the electrolysis of aqueous solution of CuSO4 by using platinum…
  41. What will happen during the electrolysis of aqueous solution of CuSO4 in the presence of…
  42. Conductivity κ is equal to ____________.
  43. Molar conductivity of ionic solution depends on ___________.
  44. For the given cell, Mg|Mg2+|| Cu2+|Cu
Multiple Choice Questions Ii
  1. The positive value of the standard electrode potential of Cu2+/Cu indicates that…
  2. e_ ^ { theta } for some half cell reactions are given below. On the basis of these…
  3. e_ ^ { theta } = 1.1 V for Daniel cell. Which of the following expressions are correct…
  4. Conductivity of an electrolytic solution depends on ____________.…
  5. ^ _{m}^{0}h_{2}o is equal to ___________.
  6. What will happen during the electrolysis of aqueous solution of CuSO4 by using platinum…
  7. What will happen during the electrolysis of aqueous solution of CuSO4 in the presence of…
  8. Conductivity κ is equal to ____________.
  9. Molar conductivity of ionic solution depends on ___________.
  10. For the given cell, Mg|Mg2+|| Cu2+|Cu
Short Answer
  1. Can absolute electrode potential of an electrode be measured?
  2. Can absolute electrode potential of an electrode be measured?
  3. Canfor cell reaction ever be equal to zero?
  4. Canfor cell reaction ever be equal to zero?
  5. Under what condition is ECell = 0 or ΔrG = 0?
  6. Under what condition is ECell = 0 or ΔrG = 0?
  7. What does the negative sign in the expression mean?
  8. What does the negative sign in the expression mean?
  9. Aqueous copper sulphate solution and aqueous silver nitrate solution are electrolysed by 1…
  10. Aqueous copper sulphate solution and aqueous silver nitrate solution are electrolysed by 1…
  11. Depict the galvanic cell in which the cell reaction is Cu + 2Ag+→ 2Ag + Cu2+…
  12. Depict the galvanic cell in which the cell reaction is Cu + 2Ag+→ 2Ag + Cu2+…
  13. Value of standard electrode potential for the oxidation of Cl– ions is more positive than…
  14. Value of standard electrode potential for the oxidation of Cl– ions is more positive than…
  15. What is electrode potential?
  16. What is electrode potential?
  17. Consider the following diagram in which an electrochemical cell is coupled toan…
  18. Consider the following diagram in which an electrochemical cell is coupled toan…
  19. Why is alternating current used for measuring the resistance of an electrolytic solution?…
  20. Why is alternating current used for measuring the resistance of an electrolytic solution?…
  21. A galvanic cell has an electrical potential of 1.1V. If an opposing potential of 1.1V is…
  22. A galvanic cell has an electrical potential of 1.1V. If an opposing potential of 1.1V is…
  23. How will the pH of brine (aq. NaCl solution) be affected when it is electrolyzed?…
  24. How will the pH of brine (aq. NaCl solution) be affected when it is electrolyzed?…
  25. Unlike dry cell, the mercury cell has a constant cell potential throughout its useful…
  26. Unlike dry cell, the mercury cell has a constant cell potential throughout its useful…
  27. Solutions of two electrolytes ‘A’ and ‘B’are diluted. The pi of ‘B’ increases 1.5 times…
  28. Solutions of two electrolytes ‘A’ and ‘B’are diluted. The of ‘B’ increases 1.5 times…
  29. When acidulated water (dil.H2SO4 solution) is electrolysed, will the pH of the solution be…
  30. When acidulated water (dil.H2SO4 solution) is electrolysed, will the pH of the solution be…
  31. In an aqueous solution, how does specific conductivity of electrolytes change with the…
  32. In an aqueous solution, how does specific conductivity of electrolytes change with the…
  33. Which reference electrode is used to measure the electrode potential of other electrodes?…
  34. Which reference electrode is used to measure the electrode potential of other electrodes?…
  35. Consider a cell given belowCu|Cu2+|| Cl—|Cl2, PtWrite the reactions that occur at anode…
  36. Consider a cell given belowCu|Cu2+|| Cl—|Cl2, PtWrite the reactions that occur at anode…
  37. Write the Nernst equation for the cell reaction in the Daniel cell. How will the ECell be…
  38. Write the Nernst equation for the cell reaction in the Daniel cell. How will the ECell be…
  39. What advantage do the fuel cells have over primary and secondary batteries?…
  40. What advantage do the fuel cells have over primary and secondary batteries?…
  41. Write the cell reaction of a lead storage battery when it is discharged. How does the…
  42. Write the cell reaction of a lead storage battery when it is discharged. How does the…
  43. Why on dilution the m_{m} of CH3COOH increases drastically, while that of CH3COONa…
  44. Why on dilution the ^ _{m} of CH3COOH increases drastically, while that of CH3COONa…
Matching Type
  1. Match the terms given in Column I with the units given in Column II.…
  2. Match the terms given in Column I with the units given in Column II.…
  3. Match the terms given in Column I with the items given in Column II.…
  4. Match the terms given in Column I with the items given in Column II.…
  5. Match the items of Column I and Column II.
  6. Match the items of Column I and Column II.
  7. Match the items of Column I and Column II.
  8. Match the items of Column I and Column II.
  9. Match the items of Column I and Column II.
  10. Match the items of Column I and Column II.
  11. Match the items of Column I and Column II on the basis of data given below:…
  12. Match the items of Column I and Column II on the basis of data given below:…
Assertion And Reason
  1. Assertion : Cu is less reactive than hydrogen.Reason : is negative…
  2. Note : In the following questions a statement of assertion followed by a statement of…
  3. Assertion : ECell should have a positive value for the cell to function.Reason : Ecathode…
  4. Note : In the following questions a statement of assertion followed by a statement of…
  5. Assertion : Conductivity of all electrolytes decreases on dilution.Reason : On dilution…
  6. Note : In the following questions a statement of assertion followed by a statement of…
  7. Assertion : cube root { pi } for weak electrolytes shows a sharp increase when the…
  8. Note : In the following questions a statement of assertion followed by a statement of…
  9. Assertion : Mercury cell does not give steady potential.Reason : In the cell reaction,…
  10. Note : In the following questions a statement of assertion followed by a statement of…
  11. Assertion : Electrolysis of NaCl solution gives chlorine at anode instead of O2.Reason :…
  12. Note : In the following questions a statement of assertion followed by a statement of…
  13. Assertion : For measuring resistance of an ionic solution an AC source is used.Reason :…
  14. Note : In the following questions a statement of assertion followed by a statement of…
  15. Assertion : Current stops flowing when ECell = 0.Reason : Equilibrium of the cell reaction…
  16. Note : In the following questions a statement of assertion followed by a statement of…
  17. Assertion : Ag+/Ag increases with increase in concentration of Ag+ ions.Reason : Ag+/Ag…
  18. Note : In the following questions a statement of assertion followed by a statement of…
  19. Note : In the following questions a statement of assertion followed by a statement of…
  20. Assertion : Copper sulphate can be stored in zinc vessel.Reason : Zinc is less reactive…
Long Answer
  1. Consider the Fig. 3.2 and answer the following questions.(i) Cell ‘A’ has ECell = 2V and…
  2. Consider the Fig. 3.2 and answer the following questions.(i) Cell ‘A’ has ECell = 2V and…
  3. Consider Fig. 3.2 and answer the questions (i) to (vi) given below.(i) Redraw the diagram…
  4. Consider Fig. 3.2 and answer the questions (i) to (vi) given below.(i) Redraw the diagram…
  5. What is the relationship between Gibbs free energy of the cell reaction in a galvanic cell…
  6. What is the relationship between Gibbs free energy of the cell reaction in a galvanic cell…

Multiple Choice Questions I
Question 1.

Which cell will measure standard electrode potential of copper electrode?
A. Pt (s) | H2 (g, 0.1 bar) | H+ (aq., 1 M) || Cu2+(aq., 1M) | Cu

B. Pt(s) | H2 (g, 1 bar) | H+ (aq., 1 M) || Cu2+ (aq., 2 M) | Cu

C. Pt(s)| H2 (g, 1 bar) | H+ (aq., 1 M) || Cu2+ (aq., 1 M) | Cu

D. Pt(s) | H2 (g, 1 bar) | H+ (aq., 0.1 M) || Cu2+ (aq., 1 M) | Cu


Answer:

When an electrode potential (which is a potential difference that develops between the electrode and electrolyte), has values of the concentrations of all the species involved in a half-cell equal to unity, it is known as standard electrode potential.


The standard hydrogen electrode which is assigned a zero potential for all temperatures to measure the potential of the given half-cell. The electrode is represented as Pt(s)|H2(g)|H+(aq) and the equation is given as H+(aq) + e-→ 1/2H2(g).


This leads to the conclusion that the pressure of hydrogen gas is 1 bar and the concentration of hydrogen ions in solution is 1 M. We place the SHE on the left and copper electrode on the right, which will help us find the standard reduction potential of copper electrode.


If the concentration of copper’s reduced and oxidized forms are at unity, then the reduction potential is equal to standard potential. So, for the half-cell equation,


Cu2+ (aq, 1M) + 2e→ Cu(s), the cell that will measure the electrode potential is


Pt(s)| H2 (g, 1 bar) | H+ (aq., 1 M) || Cu2+ (aq., 1 M) | Cu.


Thus, the correct option is (iii).


Question 2.

Which cell will measure standard electrode potential of copper electrode?
A. Pt (s) | H2 (g, 0.1 bar) | H+ (aq., 1 M) || Cu2+(aq., 1M) | Cu

B. Pt(s) | H2 (g, 1 bar) | H+ (aq., 1 M) || Cu2+ (aq., 2 M) | Cu

C. Pt(s)| H2 (g, 1 bar) | H+ (aq., 1 M) || Cu2+ (aq., 1 M) | Cu

D. Pt(s) | H2 (g, 1 bar) | H+ (aq., 0.1 M) || Cu2+ (aq., 1 M) | Cu


Answer:

When an electrode potential (which is a potential difference that develops between the electrode and electrolyte), has values of the concentrations of all the species involved in a half-cell equal to unity, it is known as standard electrode potential.


The standard hydrogen electrode which is assigned a zero potential for all temperatures to measure the potential of the given half-cell. The electrode is represented as Pt(s)|H2(g)|H+(aq) and the equation is given as H+(aq) + e-→ 1/2H2(g).


This leads to the conclusion that the pressure of hydrogen gas is 1 bar and the concentration of hydrogen ions in solution is 1 M. We place the SHE on the left and copper electrode on the right, which will help us find the standard reduction potential of copper electrode.


If the concentration of copper’s reduced and oxidized forms are at unity, then the reduction potential is equal to standard potential. So, for the half-cell equation,


Cu2+ (aq, 1M) + 2e→ Cu(s), the cell that will measure the electrode potential is


Pt(s)| H2 (g, 1 bar) | H+ (aq., 1 M) || Cu2+ (aq., 1 M) | Cu.


Thus, the correct option is (iii).


Question 3.

Electrode potential for Mg electrode varies according to the equation

. The graph of vs log [Mg2+] is

A.



B.



C.



D.




Answer:

The given equation is the Nernst equation. Here is the cell potential at a given concentration and log [Mg2+] is the log of the concentration of Mg2+. They are directly proportional to each other as increase in concentration leads to an increase in cell potential.


The given equation is equivalent to equation of straight line y = mx + c. Hence, the graph will be a straight line with as the y-intercept and a positive slope 0.059/2.


The equation is arranged in the y = mx+c form as = . The correct option is (ii).


Question 4.

Electrode potential for Mg electrode varies according to the equation

. The graph of vs log [Mg2+] is

A.



B.



C.



D.




Answer:

The given equation is the Nernst equation. Here is the cell potential at a given concentration and log [Mg2+] is the log of the concentration of Mg2+. They are directly proportional to each other as increase in concentration leads to an increase in cell potential.


The given equation is equivalent to equation of straight line y = mx + c. Hence, the graph will be a straight line with as the y-intercept and a positive slope 0.059/2.


The equation is arranged in the y = mx+c form as = . The correct option is (ii).


Question 5.

Which of the following statement is correct?
A.

B. ECell and ΔrG of cell reaction both are intensive properties.

C. ECell is an intensive property while ΔrG of cell reaction is an extensive property.

D. ECell is an extensive property while ΔrG of cell reaction is an intensive property.


Answer:

The relation between ΔrG and Ecell is given by ∆rG = –nFEcell. ∆rG is the Gibbs energy of the reaction and Ecell is the cell potential. nF is the amount of charge passed in the cell.


Ecell is an intensive property, it does not depend on the quantity of the substance, while Gibbs energy is extensive and thermodynamic property, which depends on the number of charges. Option (iii) is correct.


Question 6.

Which of the following statement is correct?
A.

B. ECell and ΔrG of cell reaction both are intensive properties.

C. ECell is an intensive property while ΔrG of cell reaction is an extensive property.

D. ECell is an extensive property while ΔrG of cell reaction is an intensive property.


Answer:

The relation between ΔrG and Ecell is given by ∆rG = –nFEcell. ∆rG is the Gibbs energy of the reaction and Ecell is the cell potential. nF is the amount of charge passed in the cell.


Ecell is an intensive property, it does not depend on the quantity of the substance, while Gibbs energy is extensive and thermodynamic property, which depends on the number of charges. Option (iii) is correct.


Question 7.

The difference between the electrode potentials of two electrodes when no current is drawn through the cell is called ___________.
A. Cell potential

B. Cell emf

C. Potential difference

D. Cell voltage


Answer:

Cell potential is defined as the potential difference between the two electrodes of a galvanic cell and it is measured in volts. It is the difference between the electrode potentials (reduction potentials) between the cathode and anode. The cell potential is known as electromotive force or emf when no current is drawn through the cell. The correct answer is (ii).


Question 8.

The difference between the electrode potentials of two electrodes when no current is drawn through the cell is called ___________.
A. Cell potential

B. Cell emf

C. Potential difference

D. Cell voltage


Answer:

Cell potential is defined as the potential difference between the two electrodes of a galvanic cell and it is measured in volts. It is the difference between the electrode potentials (reduction potentials) between the cathode and anode. The cell potential is known as electromotive force or emf when no current is drawn through the cell. The correct answer is (ii).


Question 9.

Which of the following statement is not correct about an inert electrode in a cell?
A. It does not participate in the cell reaction.

B. It provides surface either for oxidation or for reduction reaction.

C. It provides surface for conduction of electrons.

D. It provides surface for redox reaction.


Answer:

Inert electrodes are made of less reactive metals such as gold and platinum, which do not participate in the electrochemical reaction but provide a surface for the reactions to occur. Their surface is provided for either oxidation or reduction reactions or for conduction of electrons. Redox reactions, a combination of oxidation and reduction cannot occur on the surface. The correct option is (iv).


Question 10.

Which of the following statement is not correct about an inert electrode in a cell?
A. It does not participate in the cell reaction.

B. It provides surface either for oxidation or for reduction reaction.

C. It provides surface for conduction of electrons.

D. It provides surface for redox reaction.


Answer:

Inert electrodes are made of less reactive metals such as gold and platinum, which do not participate in the electrochemical reaction but provide a surface for the reactions to occur. Their surface is provided for either oxidation or reduction reactions or for conduction of electrons. Redox reactions, a combination of oxidation and reduction cannot occur on the surface. The correct option is (iv).


Question 11.

An electrochemical cell can behave like an electrolytic cell when _________.
A. Ecell = 0

B. Ecell > Eext

C. Eext > Ecell

D. Ecell = Eext


Answer:

A galvanic cell is a cell which converts chemical energy liberated in a redox reaction to electrical energy. If an externally opposite potential is applied in a slowly increasing manner to the cell, the reaction continues to occur till the potential difference is equal to the value of the cell’s potential. Then, the reaction stops and no current flows through the cell. Further increase in the potential leads to the start of the reaction again but in the opposite direction, making it function as an electrolytic cell which is a cell for using electrical energy to drive non-spontaneous chemical reactions. The correct option is (iii).


Question 12.

An electrochemical cell can behave like an electrolytic cell when ____________.
A. Ecell = 0

B. Ecell > Eext

C. Eext > Ecell

D. Ecell = Eext


Answer:

A galvanic cell is a cell which converts chemical energy liberated in a redox reaction to electrical energy. If an externally opposite potential is applied in a slowly increasing manner to the cell, the reaction continues to occur till the potential difference is equal to the value of the cell’s potential. Then, the reaction stops and no current flows through the cell. Further increase in the potential leads to the start of the reaction again but in the opposite direction, making it function as an electrolytic cell which is a cell for using electrical energy to drive non-spontaneous chemical reactions. The correct option is (iii).


Question 13.

Which of the statements about solutions of electrolytes is not correct?
A. Conductivity of solution depends upon size of ions.

B. Conductivity depends upon viscosity of solution.

C. Conductivity does not depend upon solvation of ions present in solution.

D. Conductivity of solution increases with temperature.


Answer:

When ionic substances/electrolytes are dissolved in water, they dissolve and increase the conductivity of water. The conductivity of electrolytes is dependent on various factors just like conductivity of metal solids. The various factors include nature of the electrolyte added, size of their ions produced and solvation, which is the interaction of the solute molecules with solvent, the nature of the solvent and its viscosity, concentration of the electrolyte, and temperature of the solution. Conductivity does depend on the solvation of ions due to the interaction of solute-solvent molecules. The correct answer is (iii).


Question 14.

Which of the statements about solutions of electrolytes is not correct?
A. Conductivity of solution depends upon size of ions.

B. Conductivity depends upon viscosity of solution.

C. Conductivity does not depend upon solvation of ions present in solution.

D. Conductivity of solution increases with temperature.


Answer:

When ionic substances/electrolytes are dissolved in water, they dissolve and increase the conductivity of water. The conductivity of electrolytes is dependent on various factors just like conductivity of metal solids. The various factors include nature of the electrolyte added, size of their ions produced and solvation, which is the interaction of the solute molecules with solvent, the nature of the solvent and its viscosity, concentration of the electrolyte, and temperature of the solution. Conductivity does depend on the solvation of ions due to the interaction of solute-solvent molecules. The correct answer is (iii).


Question 15.

Using the data given below find out the strongest reducing agent.



A. Cl

B. Cr

C. Cr3+

D. Mn2+


Answer:

If the standard potential of an ion is greater than zero, then the reduced form is more stable compared to hydrogen gas. On the other hand, if the standard potential is lesser than zero, then hydrogen gas is more stable than reduced form. The positive value of the standard electrode potential means that the ions of the element can oxidise hydrogen, or hydrogen ions can reduce the atoms. The negative value of the standard electrode potential indicates that hydrogen gas can oxidise the ion, or the atom can reduce hydrogen ions. So, below the value of zero, the more negative the value of the standard electrode potential, the stronger the hydrogen gas can oxidise the ion, and the stronger the ion can be as a reducing agent. Standard electrode potential of Cr is -0.74V, which is a negative value and lower than the other mentioned values, hence it is the strongest reducing agent. The correct option is (ii).


Question 16.

Using the data given below find out the strongest reducing agent.



A. Cl

B. Cr

C. Cr3+

D. Mn2+


Answer:

If the standard potential of an ion is greater than zero, then the reduced form is more stable compared to hydrogen gas. On the other hand, if the standard potential is lesser than zero, then hydrogen gas is more stable than reduced form. The positive value of the standard electrode potential means that the ions of the element can oxidise hydrogen, or hydrogen ions can reduce the atoms. The negative value of the standard electrode potential indicates that hydrogen gas can oxidise the ion, or the atom can reduce hydrogen ions. So, below the value of zero, the more negative the value of the standard electrode potential, the stronger the hydrogen gas can oxidise the ion, and the stronger the ion can be as a reducing agent. Standard electrode potential of Cr is -0.74V, which is a negative value and lower than the other mentioned values, hence it is the strongest reducing agent. The correct option is (ii).


Question 17.

Use the data given in Q.8 and find out which of the following is the strongest oxidising agent.
A. Cl

B. Mn2+

C. MnO4–

D. Cr3+


Answer:

As mentioned in the explanation of the previous question, if the standard electrode potential is higher than zero, then reduced form is more stable than the H2 gas. The positive value of the standard electrode potential means that the ions of the element can oxidise hydrogen, or hydrogen ions can reduce the atoms. The higher the positive value, the more the ability of oxidation. The equation of formation of reduced form of MnO4- is given by the equation MnO4- + 8H+ + 5e-→ Mn2+ + 2e-. The standard electrode potential is 1.51V which is highest and most positive. Option (iii) is the correct answer.


Question 18.

Use the data given in Q.8 and find out which of the following is the strongest oxidising agent.
A. Cl

B. Mn2+

C. MnO4–

D. Cr3+


Answer:

As mentioned in the explanation of the previous question, if the standard electrode potential is higher than zero, then reduced form is more stable than the H2 gas. The positive value of the standard electrode potential means that the ions of the element can oxidise hydrogen, or hydrogen ions can reduce the atoms. The higher the positive value, the more the ability of oxidation. The equation of formation of reduced form of MnO4- is given by the equation MnO4- + 8H+ + 5e-→ Mn2+ + 2e-. The standard electrode potential is 1.51V which is highest and most positive. Option (iii) is the correct answer.


Question 19.

Using the data given in Q.8 find out in which option the order of reducing power is correct.
A. Cr3+ < Cl < Mn2+ < Cr

B. Mn2+ < Cl < Cr3+ < Cr

C. Cr3+ < Cl < Cr2O72– < MnO4–

D. Mn2+ < Cr3+ < Cl < Cr


Answer:

Again referring to data and explanation of Q. 8, if the standard potential of an element is negative value, then H2 gas is more stable than the reduced form of the element, which means these elements can reduce hydrogen ions H+ ions. The more negative the value, the more its reducing power. So the order of reducing values in decreasing order is Cr > Cr3+ > Cl- > Mn2+. The correct option is (ii).


Question 20.

Using the data given in Q.8 find out in which option the order of reducing power is correct.
A. Cr3+ < Cl < Mn2+ < Cr

B. Mn2+ < Cl < Cr3+ < Cr

C. Cr3+ < Cl < Cr2O72– < MnO4–

D. Mn2+ < Cr3+ < Cl < Cr


Answer:

Again referring to data and explanation of Q. 8, if the standard potential of an element is negative value, then H2 gas is more stable than the reduced form of the element, which means these elements can reduce hydrogen ions H+ ions. The more negative the value, the more its reducing power. So the order of reducing values in decreasing order is Cr > Cr3+ > Cl- > Mn2+. The correct option is (ii).


Question 21.

Use the data given in Q.8 and find out the most stable ion in its reduced form.
A. Cl

B. Cr3+

C. Cr

D. Mn2+


Answer:

The value of standard reduction potential of Mn2+ ion is more positive than the other ions and hydrogen, hence it is most stable in its reduced form and is a strong oxidizing agent. The correct option is (iv).


Question 22.

Use the data given in Q.8 and find out the most stable ion in its reduced form.
A. Cl

B. Cr3+

C. Cr

D. Mn2+


Answer:

The value of standard reduction potential of Mn2+ ion is more positive than the other ions and hydrogen, hence it is most stable in its reduced form and is a strong oxidizing agent. The correct option is (iv).


Question 23.

Use the data of Q.8 and find out the most stable oxidised species.
A. Cr3+

B. MnO4–

C. Cr2O72–

D. Mn2+


Answer:

The value of standard reduction potential of Cr3+ ion is more negative than the other ions and hydrogen, hence it is most stable in its oxidized form and is a strong reducing agent. The correct option is (i).


Question 24.

Use the data of Q.8 and find out the most stable oxidised species.
A. Cr3+

B. MnO4–

C. Cr2O72–

D. Mn2+


Answer:

The value of standard reduction potential of Cr3+ ion is more negative than the other ions and hydrogen, hence it is most stable in its oxidized form and is a strong reducing agent. The correct option is (i).


Question 25.

The quantity of charge required to obtain one mole of aluminium from Al2O3 is ___________.
A. 1F

B. 6F

C. 3F

D. 2F


Answer:

The quantity of electricity passed in a circuit is given by the formula


Q = It


Where,


Q is the charge developed in the circuit,


I is the current in the circuit in amperes,


t is the time period for which current is allowed to pass through in the circuit.


Electrolysis of Al2O3 gives Al3+, which can be reduced to Al(s) by 3 moles of electrons. 1 Faraday or 1F is the charge on one mole of electrons.


The charge on one electron is 1.6 × 10-19C. Charge on a mole of electrons is 1.6 × 10-19 C × 6.023 × 1023 mol-1 = 96487 C mol-1.


Thus, for three moles of aluminum, 3F of charge is required. The correct option is (iii).


Question 26.

The quantity of charge required to obtain one mole of aluminium from Al2O3 is ___________.
A. 1F

B. 6F

C. 3F

D. 2F


Answer:

The quantity of electricity passed in a circuit is given by the formula


Q = It


Where,


Q is the charge developed in the circuit,


I is the current in the circuit in amperes,


t is the time period for which current is allowed to pass through in the circuit.


Electrolysis of Al2O3 gives Al3+, which can be reduced to Al(s) by 3 moles of electrons. 1 Faraday or 1F is the charge on one mole of electrons.


The charge on one electron is 1.6 × 10-19C. Charge on a mole of electrons is 1.6 × 10-19 C × 6.023 × 1023 mol-1 = 96487 C mol-1.


Thus, for three moles of aluminum, 3F of charge is required. The correct option is (iii).


Question 27.

The cell constant of a conductivity cell _____________.
A. changes with change of electrolyte.

B. changes with change of concentration of electrolyte.

C. changes with temperature of electrolyte.

D. remains constant for a cell.


Answer:

The resistivity and conductivity of an ionic solution can be measured using a conductivity cell which uses alternating current. The resistance of the solution is given by the formula R =


Where


l is the distance between the electrodes,


A is the area of cross section of cell.


The term is known as the cell constant.


It depends on the length and area of cross section of cell, hence it remains constant for a cell, regardless of nature of electrolyte.


The correct option is (iv).


Question 28.

The cell constant of a conductivity cell _____________.
A. changes with change of electrolyte.

B. changes with change of concentration of electrolyte.

C. changes with temperature of electrolyte.

D. remains constant for a cell.


Answer:

The resistivity and conductivity of an ionic solution can be measured using a conductivity cell which uses alternating current. The resistance of the solution is given by the formula R =


Where


l is the distance between the electrodes,


A is the area of cross section of cell.


The term is known as the cell constant.


It depends on the length and area of cross section of cell, hence it remains constant for a cell, regardless of nature of electrolyte.


The correct option is (iv).


Question 29.

While charging the lead storage battery ______________.
A. PbSO4 anode is reduced to Pb.

B. PbSO4 cathode is reduced to Pb.

C. PbSO4 cathode is oxidised to Pb.

D. PbSO4 anode is oxidised to PbO2.


Answer:

Lead storage battery are secondary storage cells which can be recharged after use by passing current through it in the opposite direction and can be used again. The cell reactions when the battery is in use are as follows:


Anode: Pb(s) + SO42–(aq) → PbSO4(s) + 2e


Cathode: PbO2(s) + SO42–(aq) + 4H+(aq) + 2e→ PbSO4(s) + 2H2O(l)


When the battery is charged, the reactions are reversed and PbSO4(s) on the anode and cathode is converted to Pb and PbO2. From the anodic reaction, it is seen that the reverse is reduction. The correct option is (i).


Question 30.

While charging the lead storage battery ______________.
A. PbSO4 anode is reduced to Pb.

B. PbSO4 cathode is reduced to Pb.

C. PbSO4 cathode is oxidised to Pb.

D. PbSO4 anode is oxidised to PbO2.


Answer:

Lead storage battery are secondary storage cells which can be recharged after use by passing current through it in the opposite direction and can be used again. The cell reactions when the battery is in use are as follows:


Anode: Pb(s) + SO42–(aq) → PbSO4(s) + 2e


Cathode: PbO2(s) + SO42–(aq) + 4H+(aq) + 2e→ PbSO4(s) + 2H2O(l)


When the battery is charged, the reactions are reversed and PbSO4(s) on the anode and cathode is converted to Pb and PbO2. From the anodic reaction, it is seen that the reverse is reduction. The correct option is (i).


Question 31.

is equal to __________________.
A.

B.

C.

D.


Answer:

HCl + NH4OH → NH4Cl + H2O is an acid-base neutralization reaction. We can find using Kohlrausch law of independent migration of ions. This law states that limiting molar conductivity of an electrolyte is equal to the sum of the individual limiting molar conductivities of the anion and cation of the electrolyte.


NH4OH is a weak base. We can calculate its limiting molar conductivity using limiting molar conductivities of strong salt NH4Cl and NaCl. The cation NH4+ is from strong electrolyte NH4Cl and anion is from strong base NaOH.



The correct option is (ii).


Question 32.

is equal to __________________.
A.

B.

C.

D.


Answer:

HCl + NH4OH → NH4Cl + H2O is an acid-base neutralization reaction. We can find using Kohlrausch law of independent migration of ions. This law states that limiting molar conductivity of an electrolyte is equal to the sum of the individual limiting molar conductivities of the anion and cation of the electrolyte.


NH4OH is a weak base. We can calculate its limiting molar conductivity using limiting molar conductivities of strong salt NH4Cl and NaCl. The cation NH4+ is from strong electrolyte NH4Cl and anion is from strong base NaOH.



The correct option is (ii).


Question 33.

In the electrolysis of aqueous sodium chloride solution which of the half-cell reaction will occur at anode?
A.

B.

C.

D.


Answer:

The products of molten sodium chloride and aqueous solution of sodium chloride are not the same. In molten NaCl electrolysis, the products are only Na and Cl2. In the electrolysis of aqueous NaCl, apart from Na+ and Cl- ions, H+ and OH- ions also exist because of presence of H2O. At the cathode, there is competition between two reduction reactions –


(i) Na+(aq) + e→ Na(s) where standard electrode potential Ecell = –2.71 V


(ii) H+(aq) + e → 1/2 H2(g) Ecell = 0.00 V


The reaction with higher potential value is preferred, hence the second equation takes place. But H+ is produced as a product of dissociation of H2O, which is given by H2O(l) → H+ + OH-. The net reaction at the cathode is written as H2O (l) + e→ 1/2H2 (g) + OH.


At anode, two oxidation reactions are possible.


(i) Cl (aq) → 1/2 Cl2 (g) + e Ecell = 1.36 V


(ii) 2H2O (l) → O2 (g) + 4H+ (aq) + 4e Ecell = 1.23 V


Here, the issue of overpotential arises. Some electrochemical processes which are possible at an electrode, are so slow kinetically that at lower voltages these cannot take place and an extra potential (called overpotential) needs to be applied. The reaction at anode with lower value of E cell is preferred and accordingly, water should get oxidised in preference to Cl(aq). However, because of overpotential of oxygen, second reaction is preferred. Hence, the correct option is (ii).


Question 34.

In the electrolysis of aqueous sodium chloride solution which of the half-cell reaction will occur at anode?
A.

B.

C.

D.


Answer:

The products of molten sodium chloride and aqueous solution of sodium chloride are not the same. In molten NaCl electrolysis, the products are only Na and Cl2. In the electrolysis of aqueous NaCl, apart from Na+ and Cl- ions, H+ and OH- ions also exist because of presence of H2O. At the cathode, there is competition between two reduction reactions –


(i) Na+(aq) + e→ Na(s) where standard electrode potential Ecell = –2.71 V


(ii) H+(aq) + e → 1/2 H2(g) Ecell = 0.00 V


The reaction with higher potential value is preferred, hence the second equation takes place. But H+ is produced as a product of dissociation of H2O, which is given by H2O(l) → H+ + OH-. The net reaction at the cathode is written as H2O (l) + e→ 1/2H2 (g) + OH.


At anode, two oxidation reactions are possible.


(i) Cl (aq) → 1/2 Cl2 (g) + e Ecell = 1.36 V


(ii) 2H2O (l) → O2 (g) + 4H+ (aq) + 4e Ecell = 1.23 V


Here, the issue of overpotential arises. Some electrochemical processes which are possible at an electrode, are so slow kinetically that at lower voltages these cannot take place and an extra potential (called overpotential) needs to be applied. The reaction at anode with lower value of E �cell is preferred and accordingly, water should get oxidised in preference to Cl(aq). However, because of overpotential of oxygen, second reaction is preferred. Hence, the correct option is (ii).


Question 35.

The positive value of the standard electrode potential of Cu2+/Cu indicates that ____________.
A. this redox couple is a stronger reducing agent than the H+/H2 couple.

B. this redox couple is a stronger oxidising agent than H+/H2.

C. Cu can displace H2 from acid.

D. Cu cannot displace H2 from acid.


Answer:

The standard reduction potential of Cu2+/Cu is 0.34 and is a positive value. The value lies higher than the potential of hydrogen electrode. The reduction potential value being higher than hydrogen is a stronger oxidizing agent than H+/H2. It also means that it is more stable when reduced i.e. more stable on gaining electrons. They get more easily reduced than hydrogen ions. This also means hydrogen ions cannot oxidize Cu, alternatively hydrogen gas can reduce copper ion. The reverse cannot occur, hence Cu cannot reduce H+. Options (ii) and (iv) are correct.


Question 36.

for some half cell reactions are given below. On the basis of these mark the correct answer.

(a)

(b)

(c)

A. In dilute sulphuric acid solution, hydrogen will be reduced at cathode.

B. In concentrated sulphuric acid solution, water will be oxidised at anode.

C. In dilute sulphuric acid solution, water will be oxidised at anode.

D. In dilute sulphuric acid solution, ion will be oxidised to tetrathionate ion at anode.


Answer:

In electrolysis of dilute sulphuric acid, the half reactions at cathode and anode are as follows,


Anode: 2H2O (l) → 4H+ (aq) + O2 (g) + 4e


4OH (aq) → 2H2O (l) + O2 (g) + 4e


The standard reduction potential of H2 is higher than that of SO42- ion, hence oxidation of H2O takes place instead of sulphate.


Cathode: 2H+ (aq) + 2e→ H2 (g)


Here the H+ ions are attracted to the cathode and are reduced to form H2 gas.


For concentrated sulphuric acid, there is very low amount of water present, so at the anode, the sulphate ions get attracted and forms di-anion.


2SO4⟶S2O82−+2e is the equation. Thus, at anode sulphuric acid gets oxidized, not water.


The correct options are (i) and (iii).


Question 37.

= 1.1 V for Daniel cell. Which of the following expressions are correct description of state of equilibrium in this cell?
A. 1.1 = Kc

B.

C.

D. log Kc = 1.1


Answer:

The electrode potential of any cell compared to standard hydrogen electrode is given by the Nernst equation, . Where E0 is the standard cell potential, R is the universal gas constant, T is the temperature in Kelvin, F is the Faraday constant, [Mn+] is the concentration of the electrolyte.


For a Daniell cell, the cell potential is given in the following equation after converting the natural logarithm to the base 10 and substituting the values of R, F and T = 298 K –


Ecell = - .


At equilibrium,


Ecell = 0. So, EӨcell =


Kc = [Zn2+]/[Cu2+]


So the equation can also be written as


EӨcell =


From these equations, it can be calculated that (ii) and (iii) are correct.


Question 38.

Conductivity of an electrolytic solution depends on ____________.
A. nature of electrolyte.

B. concentration of electrolyte.

C. power of AC source.

D. distance between the electrodes.


Answer:

The conductivity of a substance is the inverse of resistivity and is the conductance of a substance when it is one metre long and area of cross section is one m2. Conductivity of electrolytic solutions also can be measured in a cell and even pure water has a low level of conductivity. The conductivity of electrolytic solutions depends on the nature of the electrolyte/solute added, the size of the ions produced, their solvation, the nature of the solvent and its viscosity, the concentration of the solute and the temperature. The conductivity of a substance is constant at a constant temperature and pressure. The correct options are (i) and (ii).


Question 39.

is equal to ___________.
A.

B.

C.

D.


Answer:

We can find the solution using Kohlrausch law of independent migration of ions. This law states that limiting molar conductivity of an electrolyte is equal to the sum of the individual limiting molar conductivities of the anion and cation of the electrolyte. There are three equations mentioned in the options which are acid-base neutralization reactions. They are –


HNO3 + NaOH → NaNO3 + H2O


HCl + NaOH → NaCl + H2O


HCl + NH4OH → NH4Cl + H2O


Λom(H2O) = Λom(HNO3) + Λom(NaOH) – Λom(NaNO3)


Λom(H2O)om(HCl) + Λom(NaOH) – Λom(NaCl)


Λom(H2O)om(HCl) + Λom(NH4OH) – Λom(NH4Cl)


The correct options are (i), (iii) and (iv).


Question 40.

What will happen during the electrolysis of aqueous solution of CuSO4 by using platinum electrodes?
A. Copper will deposit at cathode.

B. Copper will deposit at anode.

C. Oxygen will be released at anode.

D. Copper will dissolve at anode.


Answer:

Dissolving CuSO4 in water will lead to dissociation of CuSO4 to Cu2+ and SO42-. Water will dissociate as H+ and OH-. On electrolysis using inert platinum electrodes, the cathode and anode reactions along with their standard electrode potential are as follows:


At anode, two reactions are possible –


(i) 2H2O (l) → 4H+ (aq) + O2 (g) + 4e OR 4OH (aq) → 2H2O (l) + O2 (g) + 4e, EӨcell = 1.23V


(ii) 2SO42- (l) → S2O82- + 2e-, EӨcell = 1.96 V


The reaction with lower electrode potential will be favoured at the anode, so O2 is released at anode.


At cathode, two reactions are possible –


(i) Cu2+ + 2e- → Cu(s), EӨcell = 0.34V


(ii) H+ + e-→ H2(g), Eocell = 0.00 V


The reaction with higher electrode potential is favoured at the cathode, so Cu metal is deposited at the anode.


The correct options are (i) and (iii).


Question 41.

What will happen during the electrolysis of aqueous solution of CuSO4 in the presence of Cu electrodes?
A. Copper will deposit at cathode.

B. Copper will dissolve at anode.

C. Oxygen will be released at anode.

D. Copper will deposit at anode.


Answer:

Dissolving CuSO4 in water will lead to dissociation of CuSO4 to Cu2+ and SO42-. Water will dissociate as H+ and OH-. In this case, copper electrodes are used instead of platinum inert electrodes. So the reactions will be different. Electrolysis of CuSO4 using Cu electrodes will deposit copper metal at the cathode and the copper at the anode dissolves to form cations in solution.


At the anode, the negatively charged ions SO42- and OH- are attracted but they are stable to exist on their own and do not get oxidized, so the copper on the anode gets oxidized to form Cu2+ ions. The reaction is Cu(s) → Cu2+ + 2e-.


At the cathode, the positively charged Cu2+ ions get attracted and get reduced to form Cu metal, which gets deposited on the copper cathode. It is favoured more than H+ to get reduced because of higher electrode potential. The reaction is given as Cu2+ + 2e- → Cu(s).


The correct options are (i) and (ii).


Question 42.

Conductivity κ is equal to ____________.
A.

B.

C.

D.


Answer:

Conductivity is the inverse of resistivity ρ, and is represented by the formula


κ =


=


= l/RA =


The quantity l/A is called cell constant denoted by the symbol, G*.


So,


κ = G*/R.


is the molar conductivity and is given by = κ/c.


Options (i) and (ii) are correct.


Question 43.

Molar conductivity of ionic solution depends on ___________.
A. temperature.

B. distance between electrodes.

C. concentration of electrolytes in solution.

D. surface area of electrodes.


Answer:

Molar conductivity of an ionic solution is defined as the conductivity of an electrolytic solution divided by the concentration of the electrolytic solution. It is represented by. Where is the conductivity and c is the concentration of the solution.


It is proportional to the resistance of the solution, hence dependent on the temperature. It is also dependent on the concentration of the solution. The correct options are (i) and (iii).


Question 44.

For the given cell, Mg|Mg2+|| Cu2+|Cu
A. Mg is cathode

B. Cu is cathode

C. The cell reaction is Mg + Cu2+→ Mg2+ + Cu

D. Cu is the oxidising agent


Answer:

According to convention, an electrochemical reaction can be represented as Reaction at anode || Reaction at cathode. The || represents the salt bridge. The reaction in question shows the anodic reaction of Mg which gets oxidized to Mg2+ and the cathodic reaction is Cu2+ which gets reduced to Cu and is deposited on the cathode. The combined half-cell reactions can be represented as


Mg → Mg2+ + 2e- at anode and Cu2+ + 2e-→ Cu(s) at cathode. Combined, it forms Mg + Cu2+→ Mg2+ + Cu.


Hence, the correct options are (ii) and (iii).



Multiple Choice Questions Ii
Question 1.

The positive value of the standard electrode potential of Cu2+/Cu indicates that ____________.
A. this redox couple is a stronger reducing agent than the H+/H2 couple.

B. this redox couple is a stronger oxidising agent than H+/H2.

C. Cu can displace H2 from acid.

D. Cu cannot displace H2 from acid.


Answer:

The standard reduction potential of Cu2+/Cu is 0.34 and is a positive value. The value lies higher than the potential of hydrogen electrode. The reduction potential value being higher than hydrogen is a stronger oxidizing agent than H+/H2. It also means that it is more stable when reduced i.e. more stable on gaining electrons. They get more easily reduced than hydrogen ions. This also means hydrogen ions cannot oxidize Cu, alternatively hydrogen gas can reduce copper ion. The reverse cannot occur, hence Cu cannot reduce H+. Options (ii) and (iv) are correct.


Question 2.

for some half cell reactions are given below. On the basis of these mark the correct answer.

(a)

(b)

(c)

A. In dilute sulphuric acid solution, hydrogen will be reduced at cathode.

B. In concentrated sulphuric acid solution, water will be oxidised at anode.

C. In dilute sulphuric acid solution, water will be oxidised at anode.

D. In dilute sulphuric acid solution, ion will be oxidised to tetrathionate ion at anode.


Answer:

In electrolysis of dilute sulphuric acid, the half reactions at cathode and anode are as follows,


Anode: 2H2O (l) → 4H+ (aq) + O2 (g) + 4e


4OH (aq) → 2H2O (l) + O2 (g) + 4e


The standard reduction potential of H2 is higher than that of SO42- ion, hence oxidation of H2O takes place instead of sulphate.


Cathode: 2H+ (aq) + 2e→ H2 (g)


Here the H+ ions are attracted to the cathode and are reduced to form H2 gas.


For concentrated sulphuric acid, there is very low amount of water present, so at the anode, the sulphate ions get attracted and forms di-anion.


2SO4⟶S2O82−+2e is the equation. Thus, at anode sulphuric acid gets oxidized, not water.


The correct options are (i) and (iii).


Question 3.

= 1.1 V for Daniel cell. Which of the following expressions are correct description of state of equilibrium in this cell?
A. 1.1 = Kc

B.

C.

D. log Kc = 1.1


Answer:

The electrode potential of any cell compared to standard hydrogen electrode is given by the Nernst equation, . Where E0 is the standard cell potential, R is the universal gas constant, T is the temperature in Kelvin, F is the Faraday constant, [Mn+] is the concentration of the electrolyte.


For a Daniell cell, the cell potential is given in the following equation after converting the natural logarithm to the base 10 and substituting the values of R, F and T = 298 K –


Ecell = - .


At equilibrium,


Ecell = 0. So, EӨcell =


Kc = [Zn2+]/[Cu2+]


So the equation can also be written as


EӨcell =


From these equations, it can be calculated that (ii) and (iii) are correct.


Question 4.

Conductivity of an electrolytic solution depends on ____________.
A. nature of electrolyte.

B. concentration of electrolyte.

C. power of AC source.

D. distance between the electrodes.


Answer:

The conductivity of a substance is the inverse of resistivity and is the conductance of a substance when it is one metre long and area of cross section is one m2. Conductivity of electrolytic solutions also can be measured in a cell and even pure water has a low level of conductivity. The conductivity of electrolytic solutions depends on the nature of the electrolyte/solute added, the size of the ions produced, their solvation, the nature of the solvent and its viscosity, the concentration of the solute and the temperature. The conductivity of a substance is constant at a constant temperature and pressure. The correct options are (i) and (ii).


Question 5.

is equal to ___________.
A.

B.

C.

D.


Answer:

We can find the solution using Kohlrausch law of independent migration of ions. This law states that limiting molar conductivity of an electrolyte is equal to the sum of the individual limiting molar conductivities of the anion and cation of the electrolyte. There are three equations mentioned in the options which are acid-base neutralization reactions. They are –


HNO3 + NaOH → NaNO3 + H2O


HCl + NaOH → NaCl + H2O


HCl + NH4OH → NH4Cl + H2O


Λom(H2O) = Λom(HNO3) + Λom(NaOH) – Λom(NaNO3)


Λom(H2O)om(HCl) + Λom(NaOH) – Λom(NaCl)


Λom(H2O)om(HCl) + Λom(NH4OH) – Λom(NH4Cl)


The correct options are (i), (iii) and (iv).


Question 6.

What will happen during the electrolysis of aqueous solution of CuSO4 by using platinum electrodes?
A. Copper will deposit at cathode.

B. Copper will deposit at anode.

C. Oxygen will be released at anode.

D. Copper will dissolve at anode.


Answer:

Dissolving CuSO4 in water will lead to dissociation of CuSO4 to Cu2+ and SO42-. Water will dissociate as H+ and OH-. On electrolysis using inert platinum electrodes, the cathode and anode reactions along with their standard electrode potential are as follows:


At anode, two reactions are possible –


(i) 2H2O (l) → 4H+ (aq) + O2 (g) + 4e OR 4OH (aq) → 2H2O (l) + O2 (g) + 4e, EӨcell = 1.23V


(ii) 2SO42- (l) → S2O82- + 2e-, EӨcell = 1.96 V


The reaction with lower electrode potential will be favoured at the anode, so O2 is released at anode.


At cathode, two reactions are possible –


(i) Cu2+ + 2e- → Cu(s), EӨcell = 0.34V


(ii) H+ + e-→ H2(g), Eocell = 0.00 V


The reaction with higher electrode potential is favoured at the cathode, so Cu metal is deposited at the anode.


The correct options are (i) and (iii).


Question 7.

What will happen during the electrolysis of aqueous solution of CuSO4 in the presence of Cu electrodes?
A. Copper will deposit at cathode.

B. Copper will dissolve at anode.

C. Oxygen will be released at anode.

D. Copper will deposit at anode.


Answer:

Dissolving CuSO4 in water will lead to dissociation of CuSO4 to Cu2+ and SO42-. Water will dissociate as H+ and OH-. In this case, copper electrodes are used instead of platinum inert electrodes. So the reactions will be different. Electrolysis of CuSO4 using Cu electrodes will deposit copper metal at the cathode and the copper at the anode dissolves to form cations in solution.


At the anode, the negatively charged ions SO42- and OH- are attracted but they are stable to exist on their own and do not get oxidized, so the copper on the anode gets oxidized to form Cu2+ ions. The reaction is Cu(s) → Cu2+ + 2e-.


At the cathode, the positively charged Cu2+ ions get attracted and get reduced to form Cu metal, which gets deposited on the copper cathode. It is favoured more than H+ to get reduced because of higher electrode potential. The reaction is given as Cu2+ + 2e- → Cu(s).


The correct options are (i) and (ii).


Question 8.

Conductivity κ is equal to ____________.
A.

B.

C.

D.


Answer:

Conductivity is the inverse of resistivity ρ, and is represented by the formula


κ =


=


= l/RA =


The quantity l/A is called cell constant denoted by the symbol, G*.


So,


κ = G*/R.


is the molar conductivity and is given by = κ/c.


Options (i) and (ii) are correct.


Question 9.

Molar conductivity of ionic solution depends on ___________.
A. temperature.

B. distance between electrodes.

C. concentration of electrolytes in solution.

D. surface area of electrodes.


Answer:

Molar conductivity of an ionic solution is defined as the conductivity of an electrolytic solution divided by the concentration of the electrolytic solution. It is represented by. Where is the conductivity and c is the concentration of the solution.


It is proportional to the resistance of the solution, hence dependent on the temperature. It is also dependent on the concentration of the solution. The correct options are (i) and (iii).


Question 10.

For the given cell, Mg|Mg2+|| Cu2+|Cu
A. Mg is cathode

B. Cu is cathode

C. The cell reaction is Mg + Cu2+→ Mg2+ + Cu

D. Cu is the oxidising agent


Answer:

According to convention, an electrochemical reaction can be represented as Reaction at anode || Reaction at cathode. The || represents the salt bridge. The reaction in question shows the anodic reaction of Mg which gets oxidized to Mg2+ and the cathodic reaction is Cu2+ which gets reduced to Cu and is deposited on the cathode. The combined half-cell reactions can be represented as


Mg → Mg2+ + 2e- at anode and Cu2+ + 2e-→ Cu(s) at cathode. Combined, it forms Mg + Cu2+→ Mg2+ + Cu.


Hence, the correct options are (ii) and (iii).



Short Answer
Question 1.

Can absolute electrode potential of an electrode be measured?


Answer:

No, the absolute electrode potential of an electrode cannot be measured independently. A half-cell reaction never occurs all by itself. Two half-cells together make an electrolytic reaction possible. We can only measure the difference in electrode potential between the two half-cells. We can also measure electrode potential difference with respect to a standard electrode.



Question 2.

Can absolute electrode potential of an electrode be measured?


Answer:

No, the absolute electrode potential of an electrode cannot be measured independently. A half-cell reaction never occurs all by itself. Two half-cells together make an electrolytic reaction possible. We can only measure the difference in electrode potential between the two half-cells. We can also measure electrode potential difference with respect to a standard electrode.



Question 3.

Canfor cell reaction ever be equal to zero?


Answer:

No, E°Cell or ∆r G° can never be equal to zero. The only standard electrode potential which is arbitrarily assigned the value zero is the standard hydrogen electrode (SHE). Since everything else is measured with respect to SHE; the E°cell can never be zero.

We know that,


r G°= -n F E°cell,


Wherer G° is the standard Gibbs energy of the reaction


n is the number of electrons transferred


F is Faraday’s constant and


E°cell is the standard electrode potential of a cell


so ∆r G° can also never be zero.


Note: There is one possibility for E°cell=0, that is when both cathode and anode are made of the same metal.



Question 4.

Canfor cell reaction ever be equal to zero?


Answer:

No, E°Cell or ∆r G° can never be equal to zero. The only standard electrode potential which is arbitrarily assigned the value zero is the standard hydrogen electrode (SHE). Since everything else is measured with respect to SHE; the E°cell can never be zero.

We know that,


r G°= -n F E°cell,


Wherer G° is the standard Gibbs energy of the reaction


n is the number of electrons transferred


F is Faraday’s constant and


E°cell is the standard electrode potential of a cell


so ∆r G° can also never be zero.


Note: There is one possibility for E°cell=0, that is when both cathode and anode are made of the same metal.



Question 5.

Under what condition is ECell = 0 or ΔrG = 0?


Answer:

Electrolysis happens when a redox reaction occurs. Like all reactions, redox reaction to moves towards equilibrium. At equilibrium condition, the cell has discharged completely, and cell potential drops to zero.

rG= -nF E cell =0


WhererG° is the standard Gibbs energy of the reaction


n is the number of electrons transferred


F is Faraday’s constant and


E°cell is the standard electrode potential of a cell



Question 6.

Under what condition is ECell = 0 or ΔrG = 0?


Answer:

Electrolysis happens when a redox reaction occurs. Like all reactions, redox reaction to moves towards equilibrium. At equilibrium condition, the cell has discharged completely, and cell potential drops to zero.

rG= -nF Ecell =0


WhererG° is the standard Gibbs energy of the reaction


n is the number of electrons transferred


F is Faraday’s constant and


E°cell is the standard electrode potential of a cell



Question 7.

What does the negative sign in the expression mean?


Answer:

By convention, all standard electrode potentials are reduction potentials. A positive value suggests that the reduced form of the species is more stable than hydrogen gas and a negative value suggests that the hydrogen gas is more stable than the reduced form of species. Here, the reduced form (Zn) is not stable. It is difficult to reduce Zn2+ to Zn. So, the reverse is more likely to happen. Zn would rather get oxidized to Zn2+ and H+ will get reduced.

Thus, Zn is more reactive than Hydrogen.



Question 8.

What does the negative sign in the expression mean?


Answer:

By convention, all standard electrode potentials are reduction potentials. A positive value suggests that the reduced form of the species is more stable than hydrogen gas and a negative value suggests that the hydrogen gas is more stable than the reduced form of species. Here, the reduced form (Zn) is not stable. It is difficult to reduce Zn2+ to Zn. So, the reverse is more likely to happen. Zn would rather get oxidized to Zn2+ and H+ will get reduced.

Thus, Zn is more reactive than Hydrogen.



Question 9.

Aqueous copper sulphate solution and aqueous silver nitrate solution are electrolysed by 1 ampere current for 10 minutes in separate electrolytic cells. Will, the mass of copper and silver, deposited on the cathode be the same or different? Explain your answer.


Answer:

Faraday’s second law states that the mass of substances deposited on the electrode is proportional to their equivalent weight if the same amount of electricity is passed through different electrolytic solutions.

Faraday’s law : M ∝ E


M=nE


Where,


M is the mass of substance deposited


n is the number of moles


E is the equivalent weight of the substance


Q=It


where,


Q is the charge


I is the current and


t is time(in seconds)


Putting the values in the above formula , we get


1A x (10 x 60) sec = 600C


The electrochemical reactions for silver and copper are as given below:


Cu2+ + 2e-→ Cu


Ag+ + e-→ Ag


1 mole of electrons have a charge equal to 1 Faraday= 96500 C


For copper, we need 2 moles of electrons or (2F of charge) and for silver only one mole of the electron (1F of charge)


So, mass deposited


For silver,


If 1 F (96500 C) or 1 mole of electrons can deposit 1 mole i.e., 108 g of Ag, then


600 C can deposit,


g = 0.671 g of silver


For copper,


If 2 Faraday(96500 C) or 2 moles of electrons can deposit 1 mole i.e., 63g of Cu, then


600 C can deposit,


g = 0.195 g of copper


The highlighted fractions represent the equivalent weight.


Equivalent mass of Cu2+ is different from the equivalent mass of Ag+ so obviously the mass of copper deposited will not be the same as the mass of silver deposited.



Question 10.

Aqueous copper sulphate solution and aqueous silver nitrate solution are electrolysed by 1 ampere current for 10 minutes in separate electrolytic cells. Will, the mass of copper and silver, deposited on the cathode be the same or different? Explain your answer.


Answer:

Faraday’s second law states that the mass of substances deposited on the electrode is proportional to their equivalent weight if the same amount of electricity is passed through different electrolytic solutions.

Faraday’s law : M ∝ E


M=nE


Where,


M is the mass of substance deposited


n is the number of moles


E is the equivalent weight of the substance


Q=It


where,


Q is the charge


I is the current and


t is time(in seconds)


Putting the values in the above formula , we get


1A x (10 x 60) sec = 600C


The electrochemical reactions for silver and copper are as given below:


Cu2+ + 2e-→ Cu


Ag+ + e-→ Ag


1 mole of electrons have a charge equal to 1 Faraday= 96500 C


For copper, we need 2 moles of electrons or (2F of charge) and for silver only one mole of the electron (1F of charge)


So, mass deposited


For silver,


If 1 F (96500 C) or 1 mole of electrons can deposit 1 mole i.e., 108 g of Ag, then


600 C can deposit,


g = 0.671 g of silver


For copper,


If 2 Faraday(96500 C) or 2 moles of electrons can deposit 1 mole i.e., 63g of Cu, then


600 C can deposit,


g = 0.195 g of copper


The highlighted fractions represent the equivalent weight.


Equivalent mass of Cu2+ is different from the equivalent mass of Ag+ so obviously the mass of copper deposited will not be the same as the mass of silver deposited.



Question 11.

Depict the galvanic cell in which the cell reaction is Cu + 2Ag+→ 2Ag + Cu2+


Answer:

Oxidation takes place at the anode and reduction takes place at the cathode.

Anode- Oxidation half-cell: Cu → Cu2+ + 2e-


Cathode- Reduction half-cell: 2Ag+ + 2e- →2Ag


Overall reaction : Cu + 2Ag+→ Cu2+ + 2Ag


By convention while representing a galvanic cell, the following steps have to be followed:


1) The anode is always placed to the left and cathode to the right


2) The metal and electrolyte of the same half-cell are separated by a vertical line


3) A double vertical line which represents the salt bridge is placed in between the two half-cells to complete the notation of the galvanic cell.


Galvanic cell notation: Cu(s)| Cu2+(aq)|| Ag+(aq)| Ag(s)



Question 12.

Depict the galvanic cell in which the cell reaction is Cu + 2Ag+→ 2Ag + Cu2+


Answer:

Oxidation takes place at the anode and reduction takes place at the cathode.

Anode- Oxidation half-cell: Cu → Cu2+ + 2e-


Cathode- Reduction half-cell: 2Ag+ + 2e- →2Ag


Overall reaction : Cu + 2Ag+→ Cu2+ + 2Ag


By convention while representing a galvanic cell, the following steps have to be followed:


1) The anode is always placed to the left and cathode to the right


2) The metal and electrolyte of the same half-cell are separated by a vertical line


3) A double vertical line which represents the salt bridge is placed in between the two half-cells to complete the notation of the galvanic cell.


Galvanic cell notation: Cu(s)| Cu2+(aq)|| Ag+(aq)| Ag(s)



Question 13.

Value of standard electrode potential for the oxidation of Cl ions is more positive than that of water, even then in the electrolysis of aqueous sodium chloride, why is Cloxidised at anode instead of water?


Answer:

A positive value for the oxidation of Cl- ions means the reduced form Cl- is more stable than Cl2 but still, oxidation of Cl- takes place.

Cl (aq) → 1/2 Cl2 (g) + e Ecell = 1.36 V ---(1)


2H2O (l) → O2 (g) + 4H+ (aq) + 4e Ecell = 1.23 V ---(2)


Reaction (2) should take place at anode instead of Reaction (1) due to lower reduction potential.


In reaction (2), the oxidation of water to oxygen is kinetically unfavourable and requires excess potential called over-potential. Over-potential is the excess potential required to drive a reaction at a particular rate.


Therefore, oxidation of water does not take place at anode.



Question 14.

Value of standard electrode potential for the oxidation of Cl ions is more positive than that of water, even then in the electrolysis of aqueous sodium chloride, why is Cloxidised at anode instead of water?


Answer:

A positive value for the oxidation of Cl- ions means the reduced form Cl- is more stable than Cl2 but still, oxidation of Cl- takes place.

Cl (aq) → 1/2 Cl2 (g) + e Ecell = 1.36 V ---(1)


2H2O (l) → O2 (g) + 4H+ (aq) + 4e Ecell = 1.23 V ---(2)


Reaction (2) should take place at anode instead of Reaction (1) due to lower reduction potential.


In reaction (2), the oxidation of water to oxygen is kinetically unfavourable and requires excess potential called over-potential. Over-potential is the excess potential required to drive a reaction at a particular rate.


Therefore, oxidation of water does not take place at anode.



Question 15.

What is electrode potential?


Answer:

The potential difference developed between an electrode and electrolyte is called the electrode potential. When the potential is measured under standard conditions, i.e. when the concentration of all species in a half-cell is unity, then it is called standard electrode potential.

By convention, the standard electrode potential is always the reduction potential.



Question 16.

What is electrode potential?


Answer:

The potential difference developed between an electrode and electrolyte is called the electrode potential. When the potential is measured under standard conditions, i.e. when the concentration of all species in a half-cell is unity, then it is called standard electrode potential.

By convention, the standard electrode potential is always the reduction potential.



Question 17.

Consider the following diagram in which an electrochemical cell is coupled toan electrolytic cell. What will be the polarity of electrodes ‘A’ and ‘B’ in the electrolytic cell?




Answer:

By convention, anode is placed on left and cathode is placed on the right. Oxidation occurs at anode and reduction occurs at the cathode.

Redox reaction for the cell is


Zn(s) + Cu2+→Zn2+(aq) + Cu(s)


Anode- oxidation half-cell: Zn(s) → Zn2+ (aq)+ 2e-


Cathode- reduction half-cell: Cu2+ (aq)+ 2e-→ Cu(s)


Zinc goes into solution as Zn2+ and leaves behind the electrons on Electrode A making it negatively charged. Cu2+ from the solution deposits on Electrode B making it positively charged.



Electrode A polarity- negative


Electrode B polarity- positive



Question 18.

Consider the following diagram in which an electrochemical cell is coupled toan electrolytic cell. What will be the polarity of electrodes ‘A’ and ‘B’ in the electrolytic cell?




Answer:

By convention, anode is placed on left and cathode is placed on the right. Oxidation occurs at anode and reduction occurs at the cathode.

Redox reaction for the cell is


Zn(s) + Cu2+→Zn2+(aq) + Cu(s)


Anode- oxidation half-cell: Zn(s) → Zn2+ (aq)+ 2e-


Cathode- reduction half-cell: Cu2+ (aq)+ 2e-→ Cu(s)


Zinc goes into solution as Zn2+ and leaves behind the electrons on Electrode A making it negatively charged. Cu2+ from the solution deposits on Electrode B making it positively charged.



Electrode A polarity- negative


Electrode B polarity- positive



Question 19.

Why is alternating current used for measuring the resistance of an electrolytic solution?


Answer:

Direct current changes the concentration of ions in solution. Alternating current stops electrolysis from happening and keeps the concentration of ions constant so that proper measurement of resistance can be made.



Question 20.

Why is alternating current used for measuring the resistance of an electrolytic solution?


Answer:

Direct current changes the concentration of ions in solution. Alternating current stops electrolysis from happening and keeps the concentration of ions constant so that proper measurement of resistance can be made.



Question 21.

A galvanic cell has an electrical potential of 1.1V. If an opposing potential of 1.1V is applied to this cell, what will happen to the cell reaction and current flowing through the cell?


Answer:

A galvanic cell has an electrical potential of 1.1V if an opposing potential of 1.1V is applied to this cell.


If the opposing potential equals the electric potential of the galvanic cell, then the flow of electrons stops.


When the reaction stops altogether, and no current flows through the cell.


Any further increase in the external potential again starts the reaction but in the opposite direction.




Question 22.

A galvanic cell has an electrical potential of 1.1V. If an opposing potential of 1.1V is applied to this cell, what will happen to the cell reaction and current flowing through the cell?


Answer:

A galvanic cell has an electrical potential of 1.1V if an opposing potential of 1.1V is applied to this cell.


If the opposing potential equals the electric potential of the galvanic cell, then the flow of electrons stops.


When the reaction stops altogether, and no current flows through the cell.


Any further increase in the external potential again starts the reaction but in the opposite direction.




Question 23.

How will the pH of brine (aq. NaCl solution) be affected when it is electrolyzed?


Answer:

NaCl (aq)→Na+ (aq) + Cl (aq)


Cathode: H2O(l ) + e→ 1/2 H2(g) + OH (aq)


Anode: Cl (aq) → 1/2 Cl2(g) + e


Overall reaction: NaCl(aq)+H2O(l)→NaOH(aq)+H2(g)+Cl2(g)


The solution contains Na+ and OH- which combine to form NaOH.


Since NaOH is a strong base. It will turn the brine solution basic, and the pH of the solution will increase.



Question 24.

How will the pH of brine (aq. NaCl solution) be affected when it is electrolyzed?


Answer:

NaCl (aq)→Na+ (aq) + Cl (aq)


Cathode: H2O(l ) + e→ 1/2 H2(g) + OH (aq)


Anode: Cl (aq) → 1/2 Cl2(g) + e


Overall reaction: NaCl(aq)+H2O(l)→NaOH(aq)+H2(g)+Cl2(g)


The solution contains Na+ and OH- which combine to form NaOH.


Since NaOH is a strong base. It will turn the brine solution basic, and the pH of the solution will increase.



Question 25.

Unlike dry cell, the mercury cell has a constant cell potential throughout its useful life. Why?


Answer:

The following diagram represents a mercury cell.



The reactions take place inside a mercury cell are as follows:


Anode: Zn(Hg) + 2OH→ ZnO(s) + H2O + 2e


Cathode: HgO + H2O + 2e→ Hg(l) + 2OH


Galvanic cell equation: Zn(Hg) + HgO(s) → ZnO(s) + Hg(l)


The above overall reaction contains no ions whose concentration can change over time. So, the cell potential remains constant throughout the mercury cell life.



Question 26.

Unlike dry cell, the mercury cell has a constant cell potential throughout its useful life. Why?


Answer:

The following diagram represents a mercury cell.



The reactions take place inside a mercury cell are as follows:


Anode: Zn(Hg) + 2OH→ ZnO(s) + H2O + 2e


Cathode: HgO + H2O + 2e→ Hg(l) + 2OH


Galvanic cell equation: Zn(Hg) + HgO(s) → ZnO(s) + Hg(l)


The above overall reaction contains no ions whose concentration can change over time. So, the cell potential remains constant throughout the mercury cell life.



Question 27.

Solutions of two electrolytes ‘A’ and ‘B’are diluted. The of ‘B’ increases 1.5 times while that of A increases 25 times. Which of the two is a strong electrolyte? Justify your answer.


Answer:

Molar conductivity of a solution at a given concentration is the conductance of volume V of a solution containing one mole of electrolyte kept between two electrodes with an area of cross section A and distance of unit length.

Molar conductivity, Λm= or Λm= κV


where


κ is the conductivity


A is the area of cross-section and


l is the distance between the electrodes.


For weak electrolytes the number of ions increases and the degree of dissociation increases with dilution and the increase is drastic. Hence, A has to be a weak electrolyte whereas B has to be a strong electrolyte because for strong electrolytes the number of ions does not change, but interionic distance decreases and therefore the increase is gradual.


Thus, electrolyte B is strong as on dilution the number of ions remains the same, only interionic attraction decreases; therefore, the increase in Λm is small.



Question 28.

Solutions of two electrolytes ‘A’ and ‘B’are diluted. The of ‘B’ increases 1.5 times while that of A increases 25 times. Which of the two is a strong electrolyte? Justify your answer.


Answer:

Molar conductivity of a solution at a given concentration is the conductance of volume V of a solution containing one mole of electrolyte kept between two electrodes with an area of cross section A and distance of unit length.

Molar conductivity, Λm= or Λm= κV


where


κ is the conductivity


A is the area of cross-section and


l is the distance between the electrodes.


For weak electrolytes the number of ions increases and the degree of dissociation increases with dilution and the increase is drastic. Hence, A has to be a weak electrolyte whereas B has to be a strong electrolyte because for strong electrolytes the number of ions does not change, but interionic distance decreases and therefore the increase is gradual.


Thus, electrolyte B is strong as on dilution the number of ions remains the same, only interionic attraction decreases; therefore, the increase in Λm is small.



Question 29.

When acidulated water (dil.H2SO4 solution) is electrolysed, will the pH of the solution be affected? Justify your answer.


Answer:

The following reactions take place when acidulated water is electrolysed.

Anode: 2H2O(l)→ O2(g) + 4H+ (aq) + 4e


Cathode: 4H+ + 4e-→ 2H2


Overall cell reaction: 2H2O(l) → O2(g) + 2H2(g)


pH remains the same because concentration of H+ ions remains constant.



Question 30.

When acidulated water (dil.H2SO4 solution) is electrolysed, will the pH of the solution be affected? Justify your answer.


Answer:

The following reactions take place when acidulated water is electrolysed.

Anode: 2H2O(l)→ O2(g) + 4H+ (aq) + 4e


Cathode: 4H+ + 4e-→ 2H2


Overall cell reaction: 2H2O(l) → O2(g) + 2H2(g)


pH remains the same because concentration of H+ ions remains constant.



Question 31.

In an aqueous solution, how does specific conductivity of electrolytes change with the addition of water?


Answer:

The conductivity of a solution by definition at any given concentration is the conductance of one unit volume of solution kept between two platinum electrodes with a unit area of cross-section and at a distance of unit length.

G=


Where


G is the conductance


κ is the conductivity


A is the area of cross-section


l is the distance between the platinum electrodes


Now, the addition of water dilutes the electrolyte. The number of ions in a given volume decreases and conductivity also reduces.



Question 32.

In an aqueous solution, how does specific conductivity of electrolytes change with the addition of water?


Answer:

The conductivity of a solution by definition at any given concentration is the conductance of one unit volume of solution kept between two platinum electrodes with a unit area of cross-section and at a distance of unit length.

G=


Where


G is the conductance


κ is the conductivity


A is the area of cross-section


l is the distance between the platinum electrodes


Now, the addition of water dilutes the electrolyte. The number of ions in a given volume decreases and conductivity also reduces.



Question 33.

Which reference electrode is used to measure the electrode potential of other electrodes?


Answer:

Standard hydrogen electrode (SHE) is used as a reference electrode. The potential of SHE is assigned the value zero. The electrode potential of other electrodes are measured with respect to this.

The standard hydrogen electrodes represented by


Pt(s)⎥ H2(g)⎥ H+ (aq)




Question 34.

Which reference electrode is used to measure the electrode potential of other electrodes?


Answer:

Standard hydrogen electrode (SHE) is used as a reference electrode. The potential of SHE is assigned the value zero. The electrode potential of other electrodes are measured with respect to this.

The standard hydrogen electrodes represented by


Pt(s)⎥ H2(g)⎥ H+ (aq)




Question 35.

Consider a cell given below

Cu|Cu2+|| Cl|Cl2, Pt

Write the reactions that occur at anode and cathode


Answer:

The oxidation takes place at the anode while the reduction takes place at the cathode.


By convention, while representing a galvanic cell, Anode is always placed to the left and cathode to the right.


Galvanic cell: Cu|Cu2+|| Cl|Cl2, Pt


Anode- Oxidation half-cell: Cu(s) → Cu2+ + 2e-


Cathode- Reduction half-cell: Cl2(g)+ 2e-→ 2Cl-


Thus, Cu is anode as it is getting oxidised.


Cl2 is cathode as it is getting reduced.



Question 36.

Consider a cell given below

Cu|Cu2+|| Cl|Cl2, Pt

Write the reactions that occur at anode and cathode


Answer:

The oxidation takes place at the anode while the reduction takes place at the cathode.


By convention, while representing a galvanic cell, Anode is always placed to the left and cathode to the right.


Galvanic cell: Cu|Cu2+|| Cl|Cl2, Pt


Anode- Oxidation half-cell: Cu(s) → Cu2+ + 2e-


Cathode- Reduction half-cell: Cl2(g)+ 2e-→ 2Cl-


Thus, Cu is anode as it is getting oxidised.


Cl2 is cathode as it is getting reduced.



Question 37.

Write the Nernst equation for the cell reaction in the Daniel cell. How will the ECell be affected when the concentration of Zn2+ ions is increased?


Answer:

Galvanic cell reaction: Zn(s)|Zn2+(aq)||Cu2+(aq)|Cu(s)


Anode- oxidation half-cell: Zn(s) → Zn2+ (aq)+ 2e-


Cathode- reduction half-cell: Cu2+ (aq)+ 2e-→ Cu(s)


Overall reaction: Zn(s) + Cu2+ (aq) → Zn2+(aq) +Cu(s)


Nernst equation:



Where, E cellis the electrode potential at any given concentration


E°cell is the standard electrode potential


R is the gas constant 8.314 JK-1mol-1


F is the Faraday constant (96500 C mol-1)


T is the temperature in Kelvin


[Zn2+] and [Cu2+] are the concentration of the Zn2+ and Cu2+ ion in the solution.


When we substitute the value for R, F and T(298K), we can reduce the equation to :




Question 38.

Write the Nernst equation for the cell reaction in the Daniel cell. How will the ECell be affected when the concentration of Zn2+ ions is increased?


Answer:

Galvanic cell reaction: Zn(s)|Zn2+(aq)||Cu2+(aq)|Cu(s)


Anode- oxidation half-cell: Zn(s) → Zn2+ (aq)+ 2e-


Cathode- reduction half-cell: Cu2+ (aq)+ 2e-→ Cu(s)


Overall reaction: Zn(s) + Cu2+ (aq) → Zn2+(aq) +Cu(s)


Nernst equation:



Where, E cellis the electrode potential at any given concentration


E°cell is the standard electrode potential


R is the gas constant 8.314 JK-1mol-1


F is the Faraday constant (96500 C mol-1)


T is the temperature in Kelvin


[Zn2+] and [Cu2+] are the concentration of the Zn2+ and Cu2+ ion in the solution.


When we substitute the value for R, F and T(298K), we can reduce the equation to :




Question 39.

What advantage do the fuel cells have over primary and secondary batteries?


Answer:

Fuel cells are cells that convert the energy of combustion of fuels like hydrogen, methanol into electrical energy. Fuel cells run continuously as long as reactants are supplied. Primary batteries discharge and are one-time use only, and secondary batteries can be recharged, but recharging takes a lot of time.

Thus, the fuel cell has scalability advantages over the primary and secondary battery.



Question 40.

What advantage do the fuel cells have over primary and secondary batteries?


Answer:

Fuel cells are cells that convert the energy of combustion of fuels like hydrogen, methanol into electrical energy. Fuel cells run continuously as long as reactants are supplied. Primary batteries discharge and are one-time use only, and secondary batteries can be recharged, but recharging takes a lot of time.

Thus, the fuel cell has scalability advantages over the primary and secondary battery.



Question 41.

Write the cell reaction of a lead storage battery when it is discharged. How does the density of the electrolyte change when the battery is discharged?


Answer:

The cell reaction of lead storage battery when it is discharged is give below:


At anode: Pb(s) + SO42–(aq) → PbSO4 (s) + 2e


At cathode: PbO2(s)+ SO42–(aq)+4H+(aq) +2e→PbSO4(s)+2H2O (l)


Overall reaction:


Pb(s)+PbO2(s)+2H2SO4(aq)→ 2PbSO4(s) + 2H2O(l)


The density is high when the concentration of H2SO4 is high, but during discharge this sulphuric acid gets consumed and becomes diluted with water. So, density decreases as the product water form dilute the H2SO4 concentration during the discharge of the battery.



Question 42.

Write the cell reaction of a lead storage battery when it is discharged. How does the density of the electrolyte change when the battery is discharged?


Answer:

The cell reaction of lead storage battery when it is discharged is give below:


At anode: Pb(s) + SO42–(aq) → PbSO4 (s) + 2e


At cathode: PbO2(s)+ SO42–(aq)+4H+(aq) +2e→PbSO4(s)+2H2O (l)


Overall reaction:


Pb(s)+PbO2(s)+2H2SO4(aq)→ 2PbSO4(s) + 2H2O(l)


The density is high when the concentration of H2SO4 is high, but during discharge this sulphuric acid gets consumed and becomes diluted with water. So, density decreases as the product water form dilute the H2SO4 concentration during the discharge of the battery.



Question 43.

Why on dilution the of CH3COOH increases drastically, while that of CH3COONa increases gradually?


Answer:

CH3COOH is a weak electrolyte and CH3COONa is a strong electrolyte. A weak electrolyte has a lower degree of dissociation at higher concentration but upon dilution, the degree of dissociation increases, the number of ions per unit volume increases and this lead to an increase in .So, the increase in drastic

For strong electrolytes upon dilution, the number of ions doesn’t change,but interionic attraction decreases and hence the increase is gradual.



Question 44.

Why on dilution the of CH3COOH increases drastically, while that of CH3COONa increases gradually?


Answer:

CH3COOH is a weak electrolyte and CH3COONa is a strong electrolyte. A weak electrolyte has a lower degree of dissociation at higher concentration but upon dilution, the degree of dissociation increases, the number of ions per unit volume increases and this lead to an increase in .So, the increase in drastic

For strong electrolytes upon dilution, the number of ions doesn’t change,but interionic attraction decreases and hence the increase is gradual.




Matching Type
Question 1.

Match the terms given in Column I with the units given in Column II.



Answer:

(i) Λm → (c) S cm2 mol-1


Here, Λm represents the molar conductivity.


Molar conductivity is expressed as Conductivity of the solution divided by its concentration.


Thus,


Λm = = = S cm2 mol-1


(ii) Ecell → (d) V


Here, Ecell represents the emf (electromotive force) of the cell and is expressed as the difference of the potential of the two half cells i.e., cathode and anode. Thus, the unit of Ecell is equalto ‘Volt’ and is denoted as ‘V’.


(iii) κ → (a) S cm-1


Here, κ (kappa) denotes the conductivity or the specific conductance of the cell.


We know, Resistance (R) = ρ =


Or, κ = G = = S cm-1


Where, ρ = Resistivity or specific resistance =


L= Distance between the electrodes = length


A= Area of cross section of the electrode


G= = Conductance


S= Siemens= SI unit of conductance


(iv) G* → (b) m-1


Here, G* represents the cell constant.


We know, G* =


Or, G*= Conductivity x Resistance = x Ω


= S m-1 x S-1 (Since, Ω = S-1)


= m-1


Where, Ω is the SI unit of resistance.



Question 2.

Match the terms given in Column I with the units given in Column II.



Answer:

(i) Λm → (c) S cm2 mol-1


Here, Λm represents the molar conductivity.


Molar conductivity is expressed as Conductivity of the solution divided by its concentration.


Thus,


Λm = = = S cm2 mol-1


(ii) Ecell → (d) V


Here, Ecell represents the emf (electromotive force) of the cell and is expressed as the difference of the potential of the two half cells i.e., cathode and anode. Thus, the unit of Ecell is equalto ‘Volt’ and is denoted as ‘V’.


(iii) κ → (a) S cm-1


Here, κ (kappa) denotes the conductivity or the specific conductance of the cell.


We know, Resistance (R) = ρ =


Or, κ = G = = S cm-1


Where, ρ = Resistivity or specific resistance =


L= Distance between the electrodes = length


A= Area of cross section of the electrode


G= = Conductance


S= Siemens= SI unit of conductance


(iv) G* → (b) m-1


Here, G* represents the cell constant.


We know, G* =


Or, G*= Conductivity x Resistance = x Ω


= S m-1 x S-1 (Since, Ω = S-1)


= m-1


Where, Ω is the SI unit of resistance.



Question 3.

Match the terms given in Column I with the items given in Column II.



Answer:

(i) Λm → (d) increases with dilution


(ii) → (a) intensive property


(iii) Κ → (b) depends on number of ions/volume


(iv) ΔrGcell → (c) extensive property


(i) Molar conductivity of the solution, Λm = = κ V


(Electrodes are at unit distance)


Where, V= volume of the solution containing 1 mole of electrolyte.


In other words, molar conductivity of the solution is directly proportional to the volume of the solution. Thus, with dilution (increase in volume of the solution), Λm increases.


(ii) Intensive property is the property of the system that does not depend of the mass of the system.


is an intensive property as it does not depend on the amount or size of the system. In fact, it depends on energy per coulomb of charge transferred.


(iii) The conductivity or the specific conductance of electrolyte depends on the number of ions per unit volume.


(iv) ΔrGcell = -nFEcell


Where, ΔrGcell is the Gibbs free energy of the reaction.


F= Faraday constant


nF = Amount of charge passed


n= number of electron transferred in the radox reaction.


Since, ΔrGcell depends on the n, it is an extensive property.



Question 4.

Match the terms given in Column I with the items given in Column II.



Answer:

(i) Λm → (d) increases with dilution


(ii) → (a) intensive property


(iii) Κ → (b) depends on number of ions/volume


(iv) ΔrGcell → (c) extensive property


(i) Molar conductivity of the solution, Λm = = κ V


(Electrodes are at unit distance)


Where, V= volume of the solution containing 1 mole of electrolyte.


In other words, molar conductivity of the solution is directly proportional to the volume of the solution. Thus, with dilution (increase in volume of the solution), Λm increases.


(ii) Intensive property is the property of the system that does not depend of the mass of the system.


is an intensive property as it does not depend on the amount or size of the system. In fact, it depends on energy per coulomb of charge transferred.


(iii) The conductivity or the specific conductance of electrolyte depends on the number of ions per unit volume.


(iv) ΔrGcell = -nFEcell


Where, ΔrGcell is the Gibbs free energy of the reaction.


F= Faraday constant


nF = Amount of charge passed


n= number of electron transferred in the radox reaction.


Since, ΔrGcell depends on the n, it is an extensive property.



Question 5.

Match the items of Column I and Column II.



Answer:

(i) Lead storage battery → (d) Pb is anode, PbO2 is cathode


(ii) Mercury cell → (c) gives steady potential


(iii) Fuel cell → (a) maximum efficiency


(iv) Rusting → (b) prevented by galvanization


(i) Lead storage battery is an example of secondary battery. It consists of a Pb anode and cathode made up of Lead dioxide.


(ii) The overall reaction of Mercury cell is represented as


Zn(Hg) + HgO(S) → ZnO(S) + Hg(l)


Mercury cell is a primary cell and the cell potential value is ~1.35V and remains constant throughout its life as they do not involve any ion whose concentration varies during its lifetime. Thus, gives a steady potential throughout its life.


(iii) Fuel cell uses Hydrogen and Oxygen to produce water molecules. The overall reaction is represented as


2H2(g) + O2(g)→ 2 H2O(l)


These cells run continuously until the reactants are depleted. They produce electricity with a higher efficiency value as compared to that by the thermal plants.


(iv) Rusting is basically the corrosion of iron in presence of air and water, where Iron is oxidized by loss of electrons to O2(g) and forms Iron oxide. But rusting can be prevented by galvanization, by applying a protective Zinc coating on the Iron.



Question 6.

Match the items of Column I and Column II.



Answer:

(i) Lead storage battery → (d) Pb is anode, PbO2 is cathode


(ii) Mercury cell → (c) gives steady potential


(iii) Fuel cell → (a) maximum efficiency


(iv) Rusting → (b) prevented by galvanization


(i) Lead storage battery is an example of secondary battery. It consists of a Pb anode and cathode made up of Lead dioxide.


(ii) The overall reaction of Mercury cell is represented as


Zn(Hg) + HgO(S) → ZnO(S) + Hg(l)


Mercury cell is a primary cell and the cell potential value is ~1.35V and remains constant throughout its life as they do not involve any ion whose concentration varies during its lifetime. Thus, gives a steady potential throughout its life.


(iii) Fuel cell uses Hydrogen and Oxygen to produce water molecules. The overall reaction is represented as


2H2(g) + O2(g)→ 2 H2O(l)


These cells run continuously until the reactants are depleted. They produce electricity with a higher efficiency value as compared to that by the thermal plants.


(iv) Rusting is basically the corrosion of iron in presence of air and water, where Iron is oxidized by loss of electrons to O2(g) and forms Iron oxide. But rusting can be prevented by galvanization, by applying a protective Zinc coating on the Iron.



Question 7.

Match the items of Column I and Column II.



Answer:

(i) K → (d)


(ii) Λm → (c)


(iii) ɑ →(b)


(iv) Q → (a) I × t


(i) We know, Resistance (R) = ρ =


Or, R κ = = G*


Or, κ=


Where, ρ = Resistivity or specific resistance =


L= Distance between the electrodes = length


A= Area of cross section of the electrode


G*= = Cell constant


(ii) Molar conductivity, Λm is defined as the conductivity of electrolyte divided by the molar concentration of the electrolyte.


Mathematically,


Λm =


Where, κ= Conductivity of the solution


C= Concentration


(iii) Degree of dissociation, α is defined as the ratio of the molar conductivity at any concentration to the limiting molar conductivity of an electrolyte.


Mathematically,


α =


(iv) Electric charge, Q is defined as the product of the electrical current passed and the duration.


Mathematically, Q = I x T



Question 8.

Match the items of Column I and Column II.



Answer:

(i) K → (d)


(ii) Λm → (c)


(iii) ɑ →(b)


(iv) Q → (a) I × t


(i) We know, Resistance (R) = ρ =


Or, R κ = = G*


Or, κ=


Where, ρ = Resistivity or specific resistance =


L= Distance between the electrodes = length


A= Area of cross section of the electrode


G*= = Cell constant


(ii) Molar conductivity, Λm is defined as the conductivity of electrolyte divided by the molar concentration of the electrolyte.


Mathematically,


Λm =


Where, κ= Conductivity of the solution


C= Concentration


(iii) Degree of dissociation, α is defined as the ratio of the molar conductivity at any concentration to the limiting molar conductivity of an electrolyte.


Mathematically,


α =


(iv) Electric charge, Q is defined as the product of the electrical current passed and the duration.


Mathematically, Q = I x T



Question 9.

Match the items of Column I and Column II.



Answer:

(i) Lechlanche cell → (d) reaction at anode, Zn → Zn2+ + 2e


(ii) Ni–Cd cell → (c) rechargeable


(iii) Fuel cell → (a) cell reaction 2H2 + O2→ 2H2O ,


(e) converts energy of combustion into electrical energy


(iv) Mercury cell → (b) does not involve any ion in solution and is used in hearing aids.


(i) Lechlanche cell consists of a Zinc anode. The chemical reaction involved at anode is represented as:


Zn(S) → Zn2+ + 2e


And the chemical reaction involved at cathode is represented as:


MnO2 + NH4+ + e-→ MnO(OH) + NH3


(ii) Ni-Cd cell is a type of secondary cell and is discharged after some time. But the good thing is that it can be charged again and therefore, they have a longer life than the lead storage cell. Thus, they are rechargeable.


(iii) Fuel cell uses Hydrogen and Oxygen to produce water molecules. The overall reaction is represented as:


2H2(g) + O2(g)→ 2 H2O(l)


It converts the energy of combustion into electrical energy. Oxygen is reduced at Cathode and the reaction involved is represented as:


O2(g) + 2H2O(l) → 4OH-(aq)


(iv) Mercury cell is a type of primary cell. The overall reaction of Mercury cell is represented as


Zn(Hg) + HgO(S) → ZnO(S) + Hg(l)


They do not involve any ion whose concentration varies during its lifetime and thus, the cell remains constant throughout its life. They are used for low current devices like in hearing aids, watches, etc.



Question 10.

Match the items of Column I and Column II.



Answer:

(i) Lechlanche cell → (d) reaction at anode, Zn → Zn2+ + 2e


(ii) Ni–Cd cell → (c) rechargeable


(iii) Fuel cell → (a) cell reaction 2H2 + O2→ 2H2O ,


(e) converts energy of combustion into electrical energy


(iv) Mercury cell → (b) does not involve any ion in solution and is used in hearing aids.


(i) Lechlanche cell consists of a Zinc anode. The chemical reaction involved at anode is represented as:


Zn(S) → Zn2+ + 2e


And the chemical reaction involved at cathode is represented as:


MnO2 + NH4+ + e-→ MnO(OH) + NH3


(ii) Ni-Cd cell is a type of secondary cell and is discharged after some time. But the good thing is that it can be charged again and therefore, they have a longer life than the lead storage cell. Thus, they are rechargeable.


(iii) Fuel cell uses Hydrogen and Oxygen to produce water molecules. The overall reaction is represented as:


2H2(g) + O2(g)→ 2 H2O(l)


It converts the energy of combustion into electrical energy. Oxygen is reduced at Cathode and the reaction involved is represented as:


O2(g) + 2H2O(l) → 4OH-(aq)


(iv) Mercury cell is a type of primary cell. The overall reaction of Mercury cell is represented as


Zn(Hg) + HgO(S) → ZnO(S) + Hg(l)


They do not involve any ion whose concentration varies during its lifetime and thus, the cell remains constant throughout its life. They are used for low current devices like in hearing aids, watches, etc.



Question 11.

Match the items of Column I and Column II on the basis of data given below:





Answer:

(i) F2 → (c) non-metal which is the best oxidizing agent


(ii) Li → (a) metal is the strongest reducing agent


(iii) Au3+ → (g) metal ion which is an oxidizing agent


(iv) Br → (e) anion that can be oxidized by Au3+


(v) Au → (d) unreactive metal


(vi) Li +→ (b) metal ion which is the weakest oxidizing agent


(vii) F → (f) anion which is the weakest reducing Agent


(i) We know, Fluoride is a non-metal and the standard electrode potential is +2.87V. High positive value means they are good oxidizing agent. Thus, F2 get easily reduced to F-.


(ii) Li+/Li have a standard electrode potential of -3.5V. Thus, this radox couple is a stronger reducing agent than the H+/H2 radox couple. In other words, they have the most negative potential value among the above chemical elements which makes Li metal a strongest reducing agent.


(iii) The standard electrode potential for Au3+/Au is + 1.4V. Thus, they are a weaker reducing agent than the H+/H2 radox couple. Positive standard electrode potential makes Au3+ a good oxidizing agent.


(iv) The standard electrode potential for this radox couple is +1.09V. Clearly, one can say that the standard electrode potential of this radox couple is less than that of Au3+/Au. Thus, Br- is weak oxidizing agent and stronger reducing agent than Au3+. Therefore, Br- anion can be oxidized by Au3+.


(v) Au3+ + 3e-→ Au(S)


Since, Au is in the solid state. Thus, its standard electrode potential will be zero. So, Au is an unreactive metal.


(vi) Li+/Li have a standard electrode potential of -3.5V. Thus, this radox couple is a stronger reducing agent than the H+/H2 radox couple. In other words, Li+ metal ion is the weakest oxidizing agent.


(vii) F2/F- have the standard electrode potential equal to +2.87V. High positive value of standard radox potential indicates that F2 is the best oxidizing agent, whereas, F- is the weakest reducing Agent.



Question 12.

Match the items of Column I and Column II on the basis of data given below:





Answer:

(i) F2 → (c) non-metal which is the best oxidizing agent


(ii) Li → (a) metal is the strongest reducing agent


(iii) Au3+ → (g) metal ion which is an oxidizing agent


(iv) Br → (e) anion that can be oxidized by Au3+


(v) Au → (d) unreactive metal


(vi) Li +→ (b) metal ion which is the weakest oxidizing agent


(vii) F → (f) anion which is the weakest reducing Agent


(i) We know, Fluoride is a non-metal and the standard electrode potential is +2.87V. High positive value means they are good oxidizing agent. Thus, F2 get easily reduced to F-.


(ii) Li+/Li have a standard electrode potential of -3.5V. Thus, this radox couple is a stronger reducing agent than the H+/H2 radox couple. In other words, they have the most negative potential value among the above chemical elements which makes Li metal a strongest reducing agent.


(iii) The standard electrode potential for Au3+/Au is + 1.4V. Thus, they are a weaker reducing agent than the H+/H2 radox couple. Positive standard electrode potential makes Au3+ a good oxidizing agent.


(iv) The standard electrode potential for this radox couple is +1.09V. Clearly, one can say that the standard electrode potential of this radox couple is less than that of Au3+/Au. Thus, Br- is weak oxidizing agent and stronger reducing agent than Au3+. Therefore, Br- anion can be oxidized by Au3+.


(v) Au3+ + 3e-→ Au(S)


Since, Au is in the solid state. Thus, its standard electrode potential will be zero. So, Au is an unreactive metal.


(vi) Li+/Li have a standard electrode potential of -3.5V. Thus, this radox couple is a stronger reducing agent than the H+/H2 radox couple. In other words, Li+ metal ion is the weakest oxidizing agent.


(vii) F2/F- have the standard electrode potential equal to +2.87V. High positive value of standard radox potential indicates that F2 is the best oxidizing agent, whereas, F- is the weakest reducing Agent.




Assertion And Reason
Question 1.

Assertion : Cu is less reactive than hydrogen.

Reason : is negative


Answer:

(iii) Assertion is true but the reason is false.

=+0.34V


= 0.00 V


Since, this radox couple have positive standard electrode potential, they are a weak reducing agent and therefore, Cu is less reactive than Hydrogen.


Hence, Cu is less reactive than Hydrogen as is positive.



Question 2.

Note : In the following questions a statement of assertion followed by a statement of reason is given. Choose the correct answer out of the following choices.

Assertion : Cu is less reactive than hydrogen.

Reason : is negative

A. Both assertion and reason are true and the reason is the correct explanation of assertion.

B. Both assertion and reason are true and the reason is not the correct explanation of assertion.

C. Assertion is true but the reason is false.

D. Both assertion and reason are false.

E. Assertion is false but reason is true.


Answer:

=+0.34V


= 0.00 V


Since, this radox couple have positive standard electrode potential, they are a weak reducing agent and therefore, Cu is less reactive than Hydrogen.


Hence, Cu is less reactive than Hydrogen as is positive.


Question 3.

Assertion : ECell should have a positive value for the cell to function.

Reason : Ecathode < Eanode


Answer:

(iii) Assertion is true but the reason is false.

Ecell should have a positive value for the cell to function. As,we know,


ΔrGcell = -nFEcell


Where, ΔrGcell = Gibbs free energy of the reaction.


For the proper functioning of the cell, the reaction must be spontaneous, i.e., ΔrGcell should have a negative value and this is true only when Ecell have a positive value.


We know, Ecell = Ecathode – Eanode


For reaction to be spontaneous, Ecell should have a positive value. This is true only when Ecathode > Eanode.



Question 4.

Note : In the following questions a statement of assertion followed by a statement of reason is given. Choose the correct answer out of the following choices.

Assertion : ECell should have a positive value for the cell to function.

Reason : Ecathode<Eanode

A. Both assertion and reason are true and the reason is the correct explanation of assertion.

B. Both assertion and reason are true and the reason is not the correct explanation of assertion.

C. Assertion is true but the reason is false.

D. Both assertion and reason are false.

E. Assertion is false but reason is true.


Answer:

Ecellshouldhave a positive value for the cell to function. As,we know,


ΔrGcell = -nFEcell


Where, ΔrGcell= Gibbs free energy of the reaction.


For the proper functioning of the cell, the reaction must be spontaneous, i.e., ΔrGcellshould have a negative value and this is true only when Ecellhave a positive value.


We know, Ecell= Ecathode – Eanode


For reaction to be spontaneous, Ecellshould have a positive value. This is true only when Ecathode>Eanode.


Question 5.

Assertion : Conductivity of all electrolytes decreases on dilution.

Reason : On dilution number of ions per unit volume decreases.


Answer:

(i) Both assertion and reason are true and the reason is the correct explanation of assertion.

Both the statements are true and the reason is also true. Conductivity of all electrolytes (both, weak and strong electrolytes) decreases on dilution as number of ions per unit volume decreases with dilution. Thus, the amount of current carried by the ions per unit volume also decreases.


Hence, Conductivity of all electrolytes decreases on dilution because with dilution, the number of ions per unit volume decreases.



Question 6.

Note : In the following questions a statement of assertion followed by a statement of reason is given. Choose the correct answer out of the following choices.

Assertion : Conductivity of all electrolytes decreases on dilution.

Reason : On dilution number of ions per unit volume decreases.

A. Both assertion and reason are true and the reason is the correct explanation of assertion.

B. Both assertion and reason are true and the reason is not the correct explanation of assertion.

C. Assertion is true but the reason is false.

D. Both assertion and reason are false.

E. Assertion is false but reason is true.


Answer:

Both the statements are true and the reason is also true. Conductivity of all electrolytes (both, weak and strong electrolytes) decreases on dilution as number of ions per unit volume decreases with dilution. Thus, the amount of current carried by the ions perunit volume also decreases.


Hence, Conductivity of all electrolytes decreases on dilution because with dilution, the number of ions per unit volume decreases.


Question 7.

Assertion : for weak electrolytes shows a sharp increase when the electrolytic solution is diluted.

Reason : For weak electrolytes degree of dissociation increases with dilution of solution.


Answer:

(i) Both assertion and reason are true and the reason is the correct explanation of assertion.

Λmfor weak electrolytes shows a sharp increase when the electrolytic solution is diluted as degree of dissociation and the number of ions in total volume of solution that contains 1 mol of electrolyte increases with dilution of solution.



Where, C= Concentration


Hence, Λm for weak electrolytes shows a sharp increase when the electrolytic solution is diluted because degree of dissociation increases with dilution of solution.



Question 8.

Note : In the following questions a statement of assertion followed by a statement of reason is given. Choose the correct answer out of the following choices.

Assertion : for weak electrolytes shows a sharp increase when the electrolytic solution is diluted.

Reason : For weak electrolytes degree of dissociation increases with dilution of solution.

A. Both assertion and reason are true and the reason is the correct explanation of assertion.

B. Both assertion and reason are true and the reason is not the correct explanation of assertion.

C. Assertion is true but the reason is false.

D. Both assertion and reason are false.

E. Assertion is false but reason is true.


Answer:

Λmfor weak electrolytes shows a sharp increase when the electrolytic solution is diluted as degree of dissociation and the number of ions in total volume of solution that contains 1 mol of electrolyte increases with dilution of solution.



Where, C= Concentration


Hence, Λmfor weak electrolytes shows a sharp increase when the electrolytic solution is diluted because degree of dissociation increases with dilution of solution.


Question 9.

Assertion : Mercury cell does not give steady potential.

Reason : In the cell reaction, ions are not involved in solution.


Answer:

(v) Assertion is false but reason is true.

The overall reaction of Mercury cell is represented as


Zn(Hg) + HgO(S) → ZnO(S) + Hg(l)


The cell potential value is ~1.35V and remains constant throughout its life as they do not involve any ion whose concentration varies during its lifetime. Thus, gives a steady potential throughout its life.



Question 10.

Note : In the following questions a statement of assertion followed by a statement of reason is given. Choose the correct answer out of the following choices.

Assertion : Mercury cell does not give steady potential.

Reason : In the cell reaction, ions are not involved in solution.

A. Both assertion and reason are true and the reason is the correct explanation of assertion.

B. Both assertion and reason are true and the reason is not the correct explanation of assertion.

C. Assertion is true but the reason is false.

D. Both assertion and reason are false.

E. Assertion is false but reason is true.


Answer:

The overall reaction of Mercury cell is represented as


Zn(Hg) + HgO(S) →ZnO(S) + Hg(l)


The cell potential value is ~1.35V and remains constant throughout its life as they do not involve any ion whose concentration varies during its lifetime. Thus, gives a steady potential throughout its life.


Question 11.

Assertion : Electrolysis of NaCl solution gives chlorine at anode instead of O2.

Reason : Formation of oxygen at anode requires overvoltage.


Answer:

(i) Both assertion and reason are true and the reason is the correct explanation of assertion.

In the electrolysis of NaCl solution, Chlorine is produced at anode instead of O2. The two half-cell reactions are:


2Cl-→ Cl2(g) + 2e- E° = -1.36V


2H2O → O2(g) + 4H+ + 4e- E° = -1.23V


Generally, the one with the lower value is preferred but in this case Chlorine is produced at anode. This is because formation of oxygen at anode requires overvoltage.



Question 12.

Note : In the following questions a statement of assertion followed by a statement of reason is given. Choose the correct answer out of the following choices.

Assertion : Electrolysis of NaCl solution gives chlorine at anode instead of O2.

Reason : Formation of oxygen at anode requires overvoltage.

A. Both assertion and reason are true and the reason is the correct explanation of assertion.

B. Both assertion and reason are true and the reason is not the correct explanation of assertion.

C. Assertion is true but the reason is false.

D. Both assertion and reason are false.

E. Assertion is false but reason is true.


Answer:

In the electrolysis of NaCl solution, Chlorine is produced at anode instead of O2. The two half-cell reactions are:


2Cl-→ Cl2(g) + 2e- E° = -1.36V


2H2O →O2(g) + 4H+ + 4e- E° = -1.23V


Generally, the one with the lower value is preferred but in this case Chlorine is produced at anode. This is because formation of oxygen at anode requires overvoltage.


Question 13.

Assertion : For measuring resistance of an ionic solution an AC source is used.

Reason : Concentration of ionic solution will change if

DC source is used.


Answer:

(i) Both assertion and reason are true and the reason is the correct explanation of assertion.

For measuring resistance of an ionic solution an AC (Alternating current) source is used because concentration of ionic solution will change if DC (Direct current) source is used. DC current can change the composition of the ionic solution.



Question 14.

Note : In the following questions a statement of assertion followed by a statement of reason is given. Choose the correct answer out of the following choices.

Assertion : For measuring resistance of an ionic solution an AC source is used.

Reason : Concentration of ionic solution will change if DC source is used.

A. Both assertion and reason are true and the reason is the correct explanation of assertion.

B. Both assertion and reason are true and the reason is not the correct explanation of assertion.

C. Assertion is true but the reason is false.

D. Both assertion and reason are false.

E. Assertion is false but reason is true.


Answer:

For measuring resistance of an ionic solution an AC (Alternating current) source is used because concentration of ionic solution will change if DC (Direct current) source is used. DC current can change the composition of the ionic solution.


Question 15.

Assertion : Current stops flowing when ECell = 0.

Reason : Equilibrium of the cell reaction is attained.


Answer:

(i) Both assertion and reason are true and the reason is the correct explanation of assertion.

We know, Ecell = Ecathode – Eanode


For reaction to be spontaneous, Ecell should have a positive value and ΔrGcell negative.


At equilibrium, Ecell = 0, as current stops flowing.


Or, Ecathode = Eanode


Hence, Current stops flowing when ECell = 0 because equilibrium of the cell reaction is attained.



Question 16.

Note : In the following questions a statement of assertion followed by a statement of reason is given. Choose the correct answer out of the following choices.

Assertion : Current stops flowing when ECell = 0.

Reason : Equilibrium of the cell reaction is attained.

A. Both assertion and reason are true and the reason is the correct explanation of assertion.

B. Both assertion and reason are true and the reason is not the correct explanation of assertion.

C. Assertion is true but the reason is false.

D. Both assertion and reason are false.

E. Assertion is false but reason is true.


Answer:

We know, Ecell= Ecathode – Eanode


For reaction to be spontaneous, Ecellshould have a positive value and ΔrGcellnegative.


At equilibrium, Ecell = 0, as current stops flowing.


Or, Ecathode = Eanode


Hence, Current stops flowing when ECell = 0 because equilibrium of the cell reaction is attained.


Question 17.

Assertion : Ag+/Ag increases with increase in concentration of Ag+ ions.

Reason : Ag+/Ag has a positive value.


Answer:

(ii) Both assertion and reason are true and the reason is not the correct explanation of assertion.


Ag+(aq) + e- → Ag(S) E° = + 0.80 V, i.e., positive radox potential


According to the Nernst equation,


At 25°C,


Ecell = - 0.059 log (For, n=1)


= - 0.059 log


(Since, molar concentration of an element in the solid state is equal to one, because the ratio of their density to their molar mass remains the same throughout the reaction)


= + 0.059 log[Ag+]


We know, for Ag+/Ag couple is fixed. Thus, with increase in Ag+ concentration, i.e., [Ag+], log[Ag+] increases. Thus, Ecell value also increases.


Hence, EAg+/Ag increases with increase in concentration of Ag+ ions.



Question 18.

Note : In the following questions a statement of assertion followed by a statement of reason is given. Choose the correct answer out of the following choices.

Assertion : Ag+/Ag increases with increase in concentration of Ag+ ions.

Reason : Ag+/Ag has a positive value.

A. Both assertion and reason are true and the reason is the correct explanation of assertion.

B. Both assertion and reason are true and the reason is not the correct explanation of assertion.

C. Assertion is true but the reason is false.

D. Both assertion and reason are false.

E. Assertion is false but reason is true.


Answer:

Ag+(aq) + e- → Ag(S) E° = + 0.80 V, i.e., positive radox potential


According to the Nernst equation,


At 25°C,


Ecell= - 0.059 log (For, n=1)


= - 0.059 log


(Since, molar concentration of an element in the solid state is equal to one, because the ratio of their density to their molar mass remains the same throughout the reaction)


= + 0.059 log[Ag+]


We know, for Ag+/Ag couple is fixed. Thus, with increase in Ag+ concentration, i.e., [Ag+], log[Ag+] increases. Thus, Ecell value also increases.


Hence, EAg+/Ag increases with increase in concentration of Ag+ ions.


Question 19.

Note : In the following questions a statement of assertion followed by a statement of reason is given. Choose the correct answer out of the following choices.

Assertion : Copper sulphate can be stored in zinc vessel.

Reason : Zinc is less reactive than copper.

A. Both assertion and reason are true and the reason is the correct explanation of assertion.

B. Both assertion and reason are true and the reason is not the correct explanation of assertion.

C. Assertion is true but the reason is false.

D. Both assertion and reason are false.

E. Assertion is false but reason is true.


Answer:

Cu2+ + 2e-→ Cu(S) E° = 0.34V


Zn2+ + 2e-→ Zn(S) E° =-0.76V


Overall reaction,


Cu2+ + Zn(S) → Cu(S) + Zn2+ ; E° = 0.34V – (-0.76V) = + 1.1V


Copper (Cu2+) sulphate can’t be stored in zinc vessel as Zinc is more reactive than Copper. Clearly, one can say that Zinc is a stronger reducing agent than Copper. Thus, Zinc donate electrons to Cu2+ and the above radox reaction occurs.


Question 20.

Assertion : Copper sulphate can be stored in zinc vessel.

Reason : Zinc is less reactive than copper.


Answer:

(iv) Both assertion and reason are false.

Cu2+ + 2e-→ Cu(S) E° = 0.34V


Zn2+ + 2e-→ Zn(S) E° = -0.76V


Overall reaction,


Cu2+ + Zn(S) → Cu(S) + Zn2+ ; E° = 0.34V – (-0.76V) = + 1.1V


Copper (Cu2+) sulphate can’t be stored in zinc vessel as Zinc is more reactive than Copper. Clearly, one can say that Zinc is a stronger reducing agent than Copper. Thus, Zinc donate electrons to Cu2+ and the above radox reaction occurs.




Long Answer
Question 1.

Consider the Fig. 3.2 and answer the following questions.



(i) Cell ‘A’ has ECell = 2V and Cell ‘B’ has ECell = 1.1V which of the two cells ‘A’ or ‘B’ will act as an electrolytic cell. Which electrode reactions will occur in this cell?

(ii) If cell ‘A’ has ECell = 0.5V and cell ‘B’ has ECell = 1.1V then what will be the reactions at anode and cathode?


Answer:

(i) Given,


ECell(A) = 2V


ECell(B) = 1.1V


• The electrolytic cell with small Ecell will act as an electrolytic cell. Therefore, Cell B with lower emf value will act as an electrolytic cell.


• The half-cell reactions are:


• Anode: Cu(S)→ Cu2+ + 2e-


• Cathode: Zn2+ + 2e-→ Zn(S)


(ii) Given,


ECell(A) = 0.5V


ECell(B) = 1.1V


• Cell B having higher emf value will behave as a galvanic cell.


• Thus, new half-cell reactions are:


• Anode: Zn(S)→ Zn2+ + 2e-


• Cathode: Cu2+ + 2e-→ Cu(S)



Question 2.

Consider the Fig. 3.2 and answer the following questions.




(i) Cell ‘A’ has ECell = 2V and Cell ‘B’ has ECell = 1.1V which of the two cells ‘A’ or ‘B’ will act as an electrolytic cell. Which electrode reactions will occur in this cell?

(ii) If cell ‘A’ has ECell = 0.5V and cell ‘B’ has ECell = 1.1V then what will be the reactions at anode and cathode?


Answer:

(i) Given,


ECell(A) = 2V


ECell(B) = 1.1V


• The electrolytic cell with small Ecell will act as an electrolytic cell. Therefore, Cell B with lower emf value will act as an electrolytic cell.


• The half-cell reactions are:


• Anode: Cu(S)→ Cu2+ + 2e-


• Cathode: Zn2+ + 2e-→ Zn(S)


(ii) Given,


ECell(A) = 0.5V


ECell(B) = 1.1V


• Cell B having higher emf value will behave as a galvanic cell.


• Thus, new half-cell reactions are:


• Anode: Zn(S)→ Zn2+ + 2e-


• Cathode: Cu2+ + 2e-→ Cu(S)



Question 3.

Consider Fig. 3.2 and answer the questions (i) to (vi) given below.

(i) Redraw the diagram to show the direction of electron flow.

(ii) Is silver plate the anode or cathode?

(iii) What will happen if salt bridge is removed?

(iv) When will the cell stop functioning?

(v) How will concentration of Zn2+ ions and Ag+ ions be affected when the cell functions?

(vi) How will the concentration of Zn2+ ions and Ag+ ions be affected after the cell becomes ‘dead’?


Answer:

(i) Here, the electrons will move from Zn to Ag.



(ii) The silver plate is Cathode, the one on the right hand side (RHS).


(iii) Salt bridge maintains the electrical neutrality and connects the two half-cells.


If the salt bridge is removed, the cell will stop working as electrons will not be able to flow from one side to another. Radox reaction will not occur.


(iv) The cell will stop functioning when equilibrium is attained. In other words, When Ecell = 0


Or, Ecathode = Eanode


(v) When cell is functioning, the concentration of Zn2+ions will increase and concentration of Ag+ ions will decrease because of the radox reaction between these two chemical species.


(vi) No change in the concentration of Zn2+ and Ag+ as the cell is dead, i.e., Ecell = 0.


According to the Nernst equation,


Ecell = - log


Therefore, = log (Ecell = 0)


We know, value is fixed at a fixed temperature, thus, the concentration of Zn2+ and Ag+ remains the same.



Question 4.

Consider Fig. 3.2 and answer the questions (i) to (vi) given below.

(i) Redraw the diagram to show the direction of electron flow.

(ii) Is silver plate the anode or cathode?

(iii) What will happen if salt bridge is removed?

(iv) When will the cell stop functioning?

(v) How will concentration of Zn2+ ions and Ag+ ions be affected when the cell functions?

(vi) How will the concentration of Zn2+ ions and Ag+ ions be affected after the cell becomes ‘dead’?


Answer:

(i) Here, the electrons will move from Zn to Ag.



(ii) The silver plate is Cathode, the one on the right hand side (RHS).


(iii) Salt bridge maintains the electrical neutrality and connects the two half-cells.


If the salt bridge is removed, the cell will stop working as electrons will not be able to flow from one side to another.Radox reaction will not occur.


(iv) The cell will stop functioning when equilibrium is attained. In other words, When Ecell= 0


Or, Ecathode = Eanode


(v) When cell is functioning, the concentration of Zn2+ions will increase and concentration of Ag+ions will decrease because of the radox reaction between these two chemical species.


(vi) No change in the concentration of Zn2+ and Ag+ as the cell is dead, i.e., Ecell = 0.


According to the Nernst equation,


Ecell= - log


Therefore, = log (Ecell = 0)


We know, value is fixed ata fixed temperature, thus, the concentrationof Zn2+ and Ag+ remains the same.



Question 5.

What is the relationship between Gibbs free energy of the cell reaction in a galvanic cell and the emf of the cell? When will the maximum work be obtained from a galvanic cell?


Answer:

• The relationship between Gibbs free energy of the cell reaction in a galvanic cell and the emf of the cell:


ΔrGcell = -nFEcell


Where, ΔrGcell= Gibbs free energy of the reaction.


F= Faraday constant


Ecell = Emf of the cell


n= Number of moles of electron transferred in the radox reaction.


• At standard condition,


When, concentration of all reacting species is unity, T= 298 K


Ecell=


Or, ΔG° = -nF


• The maximum work can be obtained from a galvanic cell when the charge is passed reversibly.


• The reversible work done by the galvanic cell is written as,


ΔrGcell = -nFEcell



Question 6.

What is the relationship between Gibbs free energy of the cell reaction in a galvanic cell and the emf of the cell? When will the maximum work be obtained from a galvanic cell?


Answer:

• The relationship between Gibbs free energy of the cell reaction in a galvanic cell and the emf of the cell:


ΔrGcell = -nFEcell


Where, ΔrGcell = Gibbs free energy of the reaction.


F= Faraday constant


Ecell = Emf of the cell


n= Number of moles of electron transferred in the radox reaction.


• At standard condition,


When, concentration of all reacting species is unity, T= 298 K


Ecell =


Or, ΔG° = -nF


• The maximum work can be obtained from a galvanic cell when the charge is passed reversibly.


• The reversible work done by the galvanic cell is written as,


ΔrGcell = -nFEcell